You are on page 1of 54

‫ملف غادة‬

‫تم ترتيبه وتصحيحه بواسطة ى‬


٢٠٢٣ ‫ شهر أكتوبر‬،‫من نرص‬

"‫نفس والشيطان‬
‫ي‬ ‫ وإن أسأت فمن‬،‫" إن أحسنت فمن هللا‬

t.me/PhMonaNasr

Regulations, Social, and Behavior

Doctor prescribed 5 mg of Drug X but pharmacist dispensed 3 mg instead.


Dispensing error
What type of error is this?
A robot for automated filling is being used in hospital, what type of error Dispending error (e.g., wrong medication,
will this reduce? dosage form or dose).
A doctor prescribes 3300 mg of MgSO4 to a patient with epilepsy, which
Prescription error
results in the baby being admitted to NICU. What type of error is this?
A patient exIperienced anaphylaxis from prescribed medication, what
Category H Error
type of error is this?
What is the category error for patient supplied amoxicillin with penicillin
Category H Error
allergy?

What is the category error of supply of one drug instead of another? Category D Error (Dispending error)

The wrong drug was prepared to be dispensed to patient, but pharmacist


found the error before medication was given to patient, what type of Category B Error
error is this?
How can this error be prevented? Double checking
A pharmacist was distracted by colleague and instead of dispensing 5mg
Environmental reconstruction
dispensed 10 mg, how should this error be prevented?
The wrong medication was given to patient, what should the manager Identify error and record/report the
do? incidence
Choice without trailing zeros, Examples of
What is the accurate way to write prescription to prevent dose errors? right answers: (0.1, 0.5), Examples of
wrong answers: (0.10, .1, .5, 0.50, 00.5)
What is the least effective way to decrease medication error? Double check by an independent employee
A nurse though a leukemia patient had been discharged and returned his
Chemotherapeutics cannot be transported
chemo medications by pneumatic tube to the pharmacy, what was the
in pneumatic tube
health violation and what should she have done?
A doctor shouted at pharmacist, but the pharmacist remained calm and
Passive behavior
did not respond, what type of behavior is this?
Pharmacist is listening to patient with crossed over arm, what type of
Closed-off
body language is this?
Adherence of particles in a dispersion form
What is the definition of flocculation?
to Create-larger-sized particles.
Which of the following local organization’s primary mission is to ensure
CBAHI
the safety and quality of healthcare is health institutions?
right dose, right patient, right drug, right
What are the ‘5 rights’?
time, right route
What does the abbreviation OU stand for? Apply to each eye (both eyes)
What does the abbreviation “gtt OU TID” mean? One drop to each eye three times a day.
A patient is requesting medication for depression, but he does not have a
prescription and he is going away on vacation for a week, what is the best Call prescriber for prescription
course of action?
Doctor wrote e-prescription for narcotic drug , what should the Return it to doctor to write a written
pharmacist do ? prescription in addition to e-scrip
Doctor wrote handwriting prescription and pharmacist was confused Call doctor to confirm what is the right
between 2 drugs, what should pharmacist do? name of drug.
Give the same active ingredient from
A patient comes to the pharmacy with a prescription, but the brand of
different company and counsel patient on
medication prescribed in unavailable, what is the best course of action?
the change.
Which medications are considered high alert medications? Chemotoxic medications
How to prevent error due to similar drug names? Tall man lettering
Which of the following method is successful in reducing medication error Implementation of computerized physician
and enhance patient safety in hospitals? order entry (CPOE) system
Who responsible for the treatment of house worker? Ministry of Health
Which hospital can house worker be treated in? General Hospital
What type of communication is writing instructions on label? Written
Omeprazole contains 30 tablets for 35 SAR. The government wants to
Reduced by 10%
reduce package size to 28 tablets. What will happen to the price?
The nurse found a different patient name on medication and returned it
to the pharmacy. The pharmacist gave it to the right patient. Which type Drug to incorrect patient error
of error?
Who is more susceptible to risk from over-the-counter medications? Child
Which factor predisposes children to medical errors? Body weight dosing
Patient has a fracture and the orthologist referred him to the rehabilitator,
Secondary to tertiary care referral
what type of classification is this?
A doctor prescribes drug for short term. What is the appropriate refill for
No refills
this prescription?
Using the tablet to inform patient on the instructions for taking their
Visual communication
medications is what type of communication?
Which of the following best described the pharmacy health literacy Measure how well the pharmacy is serving
assessment? patients with limited health literacy skills.
A pharmacist asks patient with dermatological problem questions about
Sell cosmetic mediation
her problem, what is the goal of the pharmacist?
A pharmacist wants to open pharmacy in rural area. Many pharmacists
Improve health literacy aids
have failed in this area. How can pharmacist increase chance of success?
When should a company in increase awareness of product? During sale decline of product
A pharmacist has been late 8 times to work, and policy allows for 3
consecutive days only. Which kind of behavior is considered assertive Employee warning
behavior?
A drug used to treat or prevent a rare
What does orphan drug mean?
disease.
The process of identifying the most accurate list of all medications that
the patient is taking, including name, dosage, frequency, and route, by
Medication reconciliation
comparing the medical record to an external list of medications obtained
from a patient, hospital, or other provider. What is this a definition of?
A grandmother brings her grandson who is presenting with signs of
Parent / Caregiver
poisoning, who should you get more information from?
A case about a pharmaceutical company which was shut down and asks
Advertisement of prescription only drugs
about what is the violation that caused this?
A. 0 refills
Patient suffers from short-term insomnia and is prescribed Zolpidem, how B. 1 refill
many refills should be given? C. 3 refills
D. 5 refills
A patient comes to the hospital with pain and claims to be allergic to A. Give patient opioid.
acetaminophen and to NSAIDs and requests opioids to be given for pain. B. Refuse to give opioid because patient is
Upon examination you notice signs of needle use. What should you do? a drug abuser.
A pharmacist is counselling an elderly patient prescribed anti-psychotic.
A- Write diagnosis that patient has drug
During counselling, patient says he takes from wife’s medications to help
addiction.
him sleep. The pharmacist recorded that patient has addiction because he
B- Violate the privacy of the patient
takes meds from wife. What is the mistake pharmacist made?
A- Give 5 days medication
Patient with narcotic prescription prescribed 2 days ago and is only valid B- Give 7 days medication
for 7 days. He is traveling to another country. What should you do? C- Refuse to dispense medication
D- Ask patient to get new prescription
A- Warm room
A room with temperature of 15 – 25 degree Celsius is considered? B- Cool room
C- Room temperature
A- On the table
A drug needs to be stored at temperature of 8 to -8 degree Celsius, where B- In the freezer
should it be stored? C- In the refrigerator
D- Nitrogenous container
A. Should be in separate bag
A doctor ordered methotrexate syringe and riociguat syringe and were
B. Should check label
brough from the pharmacy in the same bag. The nurse told the doctor
C. Recall all methotrexate
that methotrexate is usually yellow but the one they received is clear. D. Give the medication
What should be done?
A. Feedback
Which of the following communication techniques is the most appropriate
B. Direct contact
at the message you want to deliver to a patient (or healthcare provider) is
C. High voice tone
received correctly?
D. Non-verbal communication
A. Keep information focused and organized
Which of the following strategies are used with low literacy patients to B. Use a normal speaking tone and volume
encourage them to change their behavior? C. Use briefer educational sessions
D. Use more visual aids
A. Responsible for poison management
B. Monitor the adverse reaction reporting
Which of the following is the function of the pharmacy and therapeutic
C. Monitor the medication error reporting
committee in a hospital?
D. Responsible for drug formulary
management
A. Performing continuous education
sessions on safe dispensing
B. Providing the pharmacy with the most
Which of the following method is successful in reducing medication error updated drug references
and enhance patient safety in hospitals? C. Implementation of computerized
physician order entry (CPOE) system
D. Encourage medical staff to participate in
conferences and scientific meetings regular
A. Dispense them directly to the patient
Free medical sample drugs (formulary or non-formulary) are provided by B. Keep them with the rest of original items
pharmaceutical sales representatives to physicians or members of the C. Keep in outpatient clinics or nursing
health care team. Which of the following is the most appropriate units
recommended policy for dealing with sample drugs? D. Exclude them from in-patient pharmacy,
drug store or doctors' offices
A patient is admitted to the hospital for the treatment of atrial fibrillation.
A) Bar Coding
During hospital stay, the patient receives IV diltiazem was discharge on
B) High-Risk Drug Protocol
oral diltiazem plus his previous antihypertensive medication verapamil.
C) Medication Reconciliation
Which of the following medication error-reduction programs Would most
D) Computerized physician order entry
likely have helped to prevent this duplication of therapy?
A. This is considered as adverse drug
A patient medication was changed from carbamazepine to phenytoin. The reaction
technician pulled the 100 mg tablets of phenobarbital off instead of the 3 B. The physician should have informed the
mg tablets. The pharmacist did not notice that when the prescription was pharmacist about prescription's change
checked. The patient received a higher phenobarbital and was C. The patient should be aware of what he
hospitalized for few days for observation. Which of the following is the was prescribed and the difference
best statement concerning this situation? D. This is preventable medication error if
the pharmacy in safety measures
A. Patient's cousin who is working as out-
patient nurse in the hospital
B. Medical resident in pediatric who wants
A 25-year-old man with leukemia admitted to hematology oncology
to explore the risk leukemia
center. Who is allowed to read the patient file without additional
C. Clinical nutritionist who is assigned to
permission?
calculate the requires for the patient
D. Pharmacy resident who is planning to do
a research cell transplant
A. Health literacy
Which of the following means that individuals have the understand basic
B. Cultural competency
health information to make appropriate health?
C. Communication skills
A. Category E
A patient was dispensed citalopram instead of their medication. This led B. Category F
to patient’s hospitalisation. What type of error is this? C. Category C
D. Category B
A pharmacy is opening in a rural area with many expacts who have A. language competencies
communication issue. What is an important skill required in pharmacist? B. health literacy educational skills
A doctor prescribed the wrong drug. The pharmacist contacted the doctor
A. Ethical dilemma
to write a new prescription. The pharmacist dispensed the new corrected
B. Ethical principles (or code of ethics)
prescription to the patient without telling them what happened. Which of
C. Confidentiality
the following is related to the action of the pharmacist?
A. E-prescribing
What is the technology used in order-verification step of medication use B. code verification
process? C. point-of-care bar coding
D. Pyxis
A patient is going for Hajj for 45 days and requires Diazepam, how much A. 30 tablets
should be dispensed? B. 45 tablets
A. Narcotic pharmacist and his student
interns.
Pharmacy manager wants to restructure narcotic room, who should allow
B. nurse in charge
to enter the room?
C. physician.
D. only pharmacy “in call” and manager.
A. Nurse
Which one of the following has been mostly participating in pharmacy & B. Dietitian
therapeutic committee? C. Technician
D. Social worker
A. Pharmaceutical care
What is the broad range of health care services provided by in order to B. Medication counseling
optimize therapeutic outcomes for individual patients? C. Medication reconciliation
D. Medication therapy management
A. Tag topiramate as high alert
Pharmacist received prescription of 4 topiramate bottles of 25 mg
B. Tell patient to return the bottle
strength, the system only requires one bottle to be scanned to check if it’s
C. Tell doctor to write prescription
the right drug. The pharmacist accidentally dispensed one bottle of 100 D. Double check and visual check
mg strength. How can this error be prevented?
According to the Saudi Regulations, who is responsible for signing papers A. Hospital director
to import investigational new drugs to Saudi Arabia for use in clinical B. Chief pharmacist
trials? C. Investigative doctor
A. Director of the hospital
You caught a pharmacist stealing from the narcotics cabinet during your
B. Pharmacy chief
night shift, who is responsible for reporting this to the police?
C. Security officer
A. Saudi Food & Drug Authority
The pharmaceutical & therapeutics committee decide on which drugs are
B. Ministry of Health
added to the formulary. Who decides which brands are available in the
C. The National Unified Procurement
hospital?
Company
A. Autonomy
A pharmacist gives medication to pregnant woman despite knowing it is a B. Non-maleficence
Category X drug. What principle does violate? C. Beneficence
D. Justice
A. pregnant woman
Who should the Background, Affect, Troubling, Handling, and Empathy
B. patient with chronic disease
(BATHE) interviewing technique be used for?
C. palliative care patient
A. Stop insulin.
B. Give patient Liraglutide and tell him that
A patient with HbA1c of 11% is taking Sitagliptin, Pioglitazone, Metformin, it will help him lose weight
and Insulin. The patient fears gaining weight on insulin. What should you C. Give patient motivational interviewing to
do? address ambivalence.
D. stop insulin and increase the dose of his
oral medications.
A. Affordability
Which of the following is related to health services not reaching patients B. Acceptability
in rural area? C. Availability
D. Accessibility
A. Affordability
Which of the following is defined as the ability to obtain healthcare B. Acceptability
services in all geographical locations? C. Availability
D. Accessibility
A. Reduce the price by 25%
B. Reduce the price by 60%
What is the right action to register a new anti-hypertensive drug in KSA?
C. The price should not exceed 70%
D. Same price during the patent period
Toxicology /Side effect
What is the antidote of organophosphosphate poisoning? Atropine and Pralidoxime
A patient taking high level of methotrexate chemotherapy, what is the
Leucovorin
anti-dote?
Which molecule is the antidote of opioid? Naloxone
What is the antidote for atenolol? Glucagon
What is the antidote for gliclazide? Octreotide
What is the antidote in acetaminophen overdose? N-Acetylcysteine
What is the antidote of benzodiazepines? Flumazenil
What is the anti-dote for diazepam? Flumazenil
What is antidote of benztropine? Physostigmine
What should be used for patient with atropine-like symptoms due to TCA? Physostigmine
A patient comes to the ER with dizziness, dry mouth, and constricted
Physostigmine
pupils, what antidote should be given?
Alkalinization with Sodium Carbonate
What is the suitable antidote in Aspirin toxicity?
(increases renal excretion of aspirin)
Which sleep medication causes dose-independent withdrawal symptoms? Benzodiazepines (Z-drugs)
Which drug causes agranulocytosis? Propylthiouracil (PTU)
What is the anti-dote of iron overdose? Deferoxamine
A patient overdosed on iron supplements and was taken to the hospital
Deferoxamine
within 2 hours, what is the suitable treatment?
Which agent causes increased pupil size and cause dose dependent
Naloxone
withdrawal?
Which drug has a metallic taste? Captopril
Which medication causes voice hoarseness? Captopril
What are the side-effects of lamotrigine? Alopecia and Rash
Which drug causes sulfa allergy? Dapsone
Drug/supplement that cause heart burn? Potassium supplements
Irinotecan caused by acute cholinergic
Which drug causes severe diarrhea?
properties
Which of the following drugs induce thrombocytopenia? Unfractioned Heparin
Which drug causes thyroid toxicity? Amiodarone
Which of the following situations are most likely to be controlled by
Verapamil overdose
administering parenteral calcium as antidote?
What side-effect is cause by cyclophosphamide? Hemorrhagic cystitis
A patient experiences hypoglycemia, what is the likely causative agent? Glibenclamide (Sulphonylureas)
Which anti-diabetic agent causes hypoglycemia? Glimepiride
What should be checked for patient given Empagliflozin? Urogenital infection
What is a common side-effect of isotretinoin? Hyperlipidemia
A patient was taken to emergency due to hypoglycemia. The patient was
unaware of hypoglycemia symptoms. Which agent is responsible for Beta-blockers
masking symptoms of hypoglycemia?
Weight gain, hypothyroidism, cognitive
effects, ataxia, hand tremor, cogwheel
What is the most common side-effect of lithium therapy?
rigidity, thirst, polyuria/polydipsia, GI upset
(nausea/vomiting).
What is the drug interaction of linezolid and citalopram? Added risk of serotonin syndrome
A patient is taking medication (including daptomycin) and has elevated Daptomycin, (causes Rhabdomyolysis and
creatine phosphokinase (CPK), which drug caused this? myopathy).
What are the side-effects of amphotericin B? Nephrotoxicity
Which of the following medications may alter renal function? Gentamicin
Which antacid medication causes low magnesium levels in patient? Proton pump inhibitors
Which drug causes pulmonary embolism? Amiodarone
Inhibition of COX-1 preventing binding of
What is the interaction of ibuprofen and aspirin?
aspirin
A patient has hypertensive crisis. The patient’s medications are lisinopril,
metformin, topiramate, and phenylephrine. The patient also eats Phenylephrine (risk of serotonin syndrome)
tyramine-rich foods during lunch. What is the causative medication?
A 36-year-old patient with BMI of 32 and has been a smoker for 10 years.
Family history shows no history of strokes/MI. What is the appropriate Progesterone only pills
contraceptive?
Patient case with insomnia and patient is on multiple medication, which
Cortisone
medication is causing this?
What is the side-effect of Bleomycin? Respiratory Fibrosis
Type of interaction between diclofenac and spironolactone? Both can cause hyperkalemia
Which antibiotic causes hyperkalemia? Trimethoprim/sulfamethoxazole
A patient taking medications experienced hepatic toxicity, what is the
Trimethoprim-Sulfamethoxazole
medication responsible?
Patient experiences toxicity after eating fish, what is the cause? Mercury
Which drug is used to reduce the side-effects after completing of
Methylprednisolone
docetaxel chemotherapy?
What is the interaction of Aspirin with Indapamide? Aspirin increases absorption of Indapamide
Ataxia, slurred speech, vomiting,
What are the signs of phenytoin toxicity?
lethargy, seizures etc.
A. Arthropathy
Why are fluoroquinolones not recommended in pediatric patients?
B. Osteoporosis
A. Thiazide Diuretics
Which of the following medication can cause Angioedema as a side- B. Calcium Channel Blockers
effect? C. ACE Inhibitors
D. Angiotensin-receptor Blockers
A. Atenolol (Accumulate)
A neonate patient has hypertension. Which medication is dangerous for
B. Propanol
him?
C. Verapamil
A. Protamine sulfate
B. Sodium salicylate
What is the antidote for heparin toxicity?
C. Naloxone
D. Glucagon
A. ptosis
A 47-year-old man is admitted to hospital with atrial arrhythmia and is
B. tinnitus
prescribed amiodarone. Which of the following is the most likely side
C. Pulmonary toxicity
effect to be monitored?
D. pseudomembranous enterocolitis
A. Increase sensitivity to the sun.
What is a common side-effect of rifampicin?
B. Orange-colored urine
Basics
How many natural amino acids in the human body? 20 natural amino acids
Which amino acids are conjugated in the liver? Glycine & Taurine
Cysteine, Lysine, Arginine, Methionine
Which amino acids are involved in production of keratin (nail&hair)?
(CLAM)
Isoleucine, Lysine, Leucine, Valine,
Which of the following is an essential amino acid? Methionine, Phenylalanine, Tryptophan,
Threonine, Histidine, Arginine
How many ATP molecules are produced from 1 glucose molecule? 38
How many calories in 10 mg of lipid? 90 kcal
Which carbohydrates are pentose sugars? Ribose
Stimulate the secretion of bicarbonate-rich
What is the role of secretin?
pancreatic fluid.
What are the components of fungal cell wall? Chitin+Glucans+ Glycoproteins
What is a small, circular, double-stranded DNA that is distinct from a cell’s
Plasmid
chromosomal DNA known as?
Which of these is contains an inhaled cortisone? Pulmicort, Symbicort
What are the secondary lymphoid organs? Tonsils, Spleen, and Lymph nodes
Which surface protein is expressed by T helper (Th) cells? CD4
What type of hypersensitivity reaction is hemolytic anemia? Type 2 hypersensitivity
What Immunoglobulin is involved in allergic asthma? Immunoglobulin E
Which immunoglobulin is able to pass through placenta? Immunoglobulin G
Which immunoglobulin is involved in passive immunity? Immunoglobulin G
Variable region of light and heavy chain
Which part of immunoglobulin interacts with antigen? (known as Fab: fragment antigen-binding
region)
Which part of immunoglobulin attaches to B-cell? Heavy chain
An antibody which binds to antigen or
What is an opsonin? microorganism making them more
susceptible to phagocytosis.
Parasitic infection increases which cell type? Eosinophils
Which cell is responsible for Type I hypersensitivity reactions? Mast cells
Interleukin-10 (not sure about this but only
Which of the following has an inhibitory effect on mast cells?
information I found)
Which immunoglobulin IG activates mast cells? Immunoglobulin E
What is a 2nd mediator during hypersensitivity reactions? Leukotriene
Which bacteria causes Plague disease? Yersinia pestis
Bacteria which cause meningitis? Neisseria meningitis.
Which gram-positive bacteria causes respiratory disease? Streptococcus pneumonia
Which bacteria is a Catalase positive & coagulase positive bacterium? Staphylococcus aureus
Identification of staphylococcus strain.
What is coagulase test used for? (Coagulase positive is staphylococcus
aureus)
Which test can be used to differentiation between P. aeruginosa and E.
Lactose fermentation test
coli?
Catalyze the nucleophilic carbonyl
What is the function of the bacterial transpeptidase enzyme? substitution process required for cross-
linking peptidoglycan in bacterial cell walls.
Source of cryptococcus neoforms? Pigeon droppings
Metabolism of Catecholamines? Rapid COMT
Which one is secondary messenger? Cyclic AMP
1,4,5-triphosphate (IP-3) and diacylglycerol
Which one causes release of intracellular calcium?
(DAG)
What is the role of mosquito in transmission of malaria? Vector of malaria
Type of interaction between smoking and theophylline? Smoking decreases levels of theophylline
Which lymphocyte is responsible for production of immunoglobulins? B- lymphocyte
Genetic information in virus? DNA or RNA
DNA ligase is the enzyme for ligation of? 2 DNA strands
TB skin test belongs to which type of hypersensitivity reaction? Delayed hypersensitivity
Which hormones inhibit gluconeogenesis? Insulin
They are involved in synthetic reactions
What is the function of the synthase enzyme?
and catalyze the joining of two molecules.
Where is T-cells developed and matured but not activated? Thymus
Proteins that recognize molecules
found on pathogens. The activation of
What is the pattern receptor recognition? pattern recognition receptors is
responsible for initiation of innate
immunity.
hich major histocompatibility complex (MHC) molecule does T helper
MHC 2
(CD4+) cells bind to?
Which major histocompatibility complex (MHC) molecule does cytotoxic
MHC 1
(CD8+) T cells bind to?
A. Arginine
B. Cysteine
Which amino acid is acidic?
C. Alanine
D. Methionine
A. DNA ligase
Which enzyme is used by bacteria to protect it from incorporation of
B. Endonuclease
foreign genetic material?
C. RNA polymerase
A. Lysine
Which C-terminus amino acid does trypsin cleave? B. Leucine
D.Phenylalanine
A- Transcription
B- Translation
Which one of the following is the first step in the central dogma?
C- Replication
D- Splicing
A- Give the meds for the remaining days (3
Patient has a narcotic prescription from Riyadh and went to a pharmacy in
days).
Dammam. The medication is for pain after major surgery and it’s for 5
B- Call physician
days, but the patient came after 2 days from the prescription date?
C- Don’t give
A patient comes to the pharmacy and tells the pharmacist he is a loyal A- Tell him that you gave him the wrong
customer to the pharmacy and likes the pharmacist. 2 days later, the information.
patient comes to the pharmacy angry and shouting because he says the B- Tell him to leave the pharmacy
pharmacist gave him the wrong information. What should the pharmacist C- Apologize to patient
do? D- Ask him what the wrong information is?
A. Glucagon
B. Glucose
Which of the following inhibits insulin?
C. Epinephrine
D. Caortisol
A. Immune-complex hypersensitivity
An 8-year-old child had difficulty in breathing, swelling of tips and tongue,
B. Delayed hypersensitivity
and fainted following eating a piece of cake containing nuts. Which of the
C. Cytotoxic hypersensitivity
following types of hypersensitivity reactions the child is exhibiting?
D. Immediate hypersensitivity
A. Human
B. Humanized
What type of antibiotic is Adalimumab?
C. Murine
D. Chimeric
A. protein from the same patient's body
Which one will generate the biggest immune response? B. bacteria toxins
C. Cholesterol from a different person
D. polysaccharide molecule from natural
source
A. Type I hypersensitivity
B. Type II hypersensitivity
In which hypersensitivity reaction is IgG produced?
C. Type III hypersensitivity
D. Type IV hypersensitivity
Patient came to ER complaining from dysuria, burning sensation when
A- E.coli
urinating, microbiology lab results revealed presence of Gram-negative
B- Neisseria meningitidis
rod bacteria, which is the likely causative bacteria?
A. Staphylococcus
Purple-stained bacteria under the microscope and is diplococcus. What is
B. Streptococcus
this bacterium?
C. Neisseria Meningitis
A. Tetanus vaccine increases activity of
diphtheria vaccine.
B. Tetanus vaccine decreases activity of
diphtheria vaccine.
What is correct regarding tetanus?
C. Tetanus is exclusively environmental
born pathogen.
D. Tetanus is both environmental and
transmitted pathogen.
A. Skin
What is the largest lymphoid organ? B. Liver
C. Spleen
Vaccinations
Who is responsible for vaccination in the KSA? Ministry of Health
Which vaccine should be avoided in patient with egg allergy? influenza and yellow fever vaccines
A patient is travelling to a country with outbreaks of salmonella infection, Typhoid vaccine (injection >2 weeks before
what should the patient receive before travelling? travel, oral >1 week before travel).
Influenza-vaccine-and pneumococcal
What vaccines are required for patient with Asthma/COPD?
polysaccharide vaccine
No, need to wait 48hrs after stopping
Can a baby receiving antiviral drug take live vaccine?
antiviral to receive vaccine
No,varicella is live vaccine thus must wait 2
Can a patient who is taking Acyclovir receive the Varicella vaccination?
days after d/c acyclovir
A 65 yo patient is going on summer vacation, his vaccination record shows
that he receives influenza vaccine every year and PCV20 4 years, what Hepatitis A
vaccination should he be given?
No, give vaccine 3 months after
Can vaccine and immunoglobulins be given together?
immunoglobulin.
A patient came to take MMR vaccine and has taken immunoglobulin for Give after 2 months (need 3 months
Hep A last month. What should be done? between vaccine and immunoglobulin)
What is the best treatment for patient with a deep wound and with
Td vaccination and immunoglobulin
unknown Td history?
A patient has hypertension, diabetes, and rheumatoid arthritis. The
patient is taking anti-diabetic and anti-hypertensive medications. Which Rheumatoid clinic
clinic should the patient be referred to?
A patient is taking hydrochlorothiazide and Eplerenone (and other
medications). hydrochlorothiazide causes hypokalemia. Which of the Eplerenone (potassium sparing diuretic)
patient’s medication will correct this?
Which vaccinations are required for COPD patient? Pneumococcal and influenza vaccine
What vaccinations are required for hajj? Influenza and meningococcal
A child is taking amoxicillin for 7 days and his vaccination is due, what is
Give vaccine
the best action?
When is MMR vaccine taken? 12 months
A. Nothing
A nurse in contact with blood of Hep B infected patient. Her serology test
B. Give vaccine alone.
shows negative Hep B surface antigen and negative antibody to Hep B.
C. immunoglobulin alone
What treatment should be given?
D. Vaccine with Immunoglobulin
A- Patient does not need another dose
A 42-year-old patient with asthma has received pneumococcal vaccine 4 until he is 65 years old.
years ago and asks if her needs to take it again. What is the suitable B- Administer PPV23
action? C- Administer PCV13.
D- Another dose is not needed.
A. Zoster
B. Varicella
What is the vaccine for shingles?
C. BCG
D. MMR
A. Influenza vaccine
A 20-year-old female has received all immunization as a child (last
B. Hepatitis A vaccine
immunization at 11 years old), which vaccine should she receive?
C. Pneumococcal vaccine
Economy
A pharmacist is comparing the clinical outcome of two drugs with
Cost-effectiveness analysis (CEA)
different costs. Which type of pharmacoeconomic analysis is this?
What type of pharmacoeconomic analysis compares outcomes of two
Cost Benefit Analysis
interventions in monetary values?
A pharmacist is comparing the cost of two generic drugs, what type of
Cost minimization analysis
pharmacoeconomic analysis is this?
What is an important factor when comparing two drugs using cost-
Must have equivalent effectiveness
minimization analysis?
What is is the net cost divided by changes in health outcomes? Cost-effectiveness ratio
What does a QALY of 1 mean? 1 year in perfect health
What is the measurement unit of a CUA? Quality-adjusted Life Years (QALY)
What type of cost is hospital meals? Direct non-medical cost
What type of cost is loss of productivity due to illness? Indirect cost
What type of healthcare cost category is morbidity and mortality? Indirect medical cost
What type of cost is hotel price and travel fees? Direct non-medical cost
What type of costs is traveling expenses from Jeddah to Riyadh to receive
Direct non-medical cost
health care from a pharmacoeconomic perspective?
What type of healthcare cost category is home medical visits? Direct medical cost
What type of cost is emotional distress? Intangible cost
107,000 (free cash flow = profit –
A company made a profit of 120,000, the operating expenses were 5,000,
[operating expenses + tax/zakah + other
and Zakah was 8,000, what is the free cash flow?
expenses])
Patient is non-adherent and after questioning appears to be unable to pay
Cause: Economic. Solution: talk to patient
for medication due to insurance running out, what is the cause of non-
about benefits and risks
adherence and what is the best course of action?
Individuals have the capacity to understand
What is the definition of health literacy? basic health information to make
appropriate health decisions.
A manager asked a pharmacist to make a promotion for the pharmacy,
Marketing
what is this called?
What is a long-term action plan called? Strategic plan
Annual consumption/sales value (annual
Which factor affects the inventory ABC classification?
number of units sold x cost per unit
A hospital has 100 available beds, and they have 3 ambulatory care clinics.
Strength (beds are internal resources
They want to start an ambulatory care service. Which one of the SWOT
which hospital has)
analysis categories is this?
Which of the following committees is responsible for requests of adding
Pharmacy & Therapeutics Committee
or removing a drug from hospital formulary?
The label on medication is different to the product in the package. Who Quality department of the pharmaceutical
should you report this to? company
Two new DM drug both of them reduce HbA1c by 1.5%, drug A cost 5,300
SAR annually and drug B cost 3000 SAR annually, the only DM drug also
Reject drug A and except Drug B
reduce HbA1c by 1.5%, and cost 3500 SAR annually. Which of the
following is appropriate?
A pharmacy has been suffering losses for 5 years, what is the best strategy
SWAT analysis
that the new employee should do to improve situation?
Inventory turnover = COGS / Average
Calculating inventory turnover ratio?
inventory value
A. Cost of goods sold
What type of cost is raw materials? B. Liability
C. Asset
A- Accept both drugs
A pharmacy already has medication for 3500 SAR but has side-effects.
B- Reject both drugs
There are two alternative drugs with the same efficacy, drug A costs 3000
C- Accept A and reject B
SAR and drug B costs 5000 SAR, what is the best decision?
D- Reject A and accept B
A- Cost-effectiveness analysis
Two medications with different costs and different effects, what type of B- Cost-minimization analysis
pharmacoeconomic analysis should be conducted? C- Cost-utility analysis
D- Cost-benefit analysis
A-Replace with Rivastigmine because
Physostigmine produce amphetamine.
B-Replace with Physostigmine because
Rivastigmine produce amphetamine.
If there is a discussion between choosing physostigmine and rivastigmine?
C-Replace with Physostigmine because has
lower SE
D-Replace with Rivastigmine because has
lower SE.
A. 3
In a total population of 1000, there are only three deaths. What is the B. 300
mortality? C. 0.003
D. 3000
A- Relative risk ratio
What measure is used for Cross-sectional study?
B- Odds ratio
A. Randomized Controlled Trials
Which study is used to establish the causation between variables? B. Cross-sectional Studies
C. Cohort Studies
A- Randomized Controlled Trials
Which study uses questionnaire? B- Cross-sectional Studies
C- Cohort Studies
A researcher wants to assess the role of estrogen in postmenopausal
A- Cohort study
women by giving estrogen to patients. What type of study is suitable for
B- Randomized Controlled Trials
this?
A- Prospective cohort study
Which type of Study used to follow-up patient?
B- Retrospective cohort study
A. time trade off
B. Willingness to pay / Human capital
A pharmacist was asked to conduct cost-benefit analysis, what measure is
approach
used to assess benefit?
C. health utility index
D. standard gamble
A. SOAP
Old pharmacy has not been making profit for the past 5 years. A new
B. SWOT
manager wants to make it profitable. What strategy will help him achieve
C. Process Description
that?
D. Data Modeling
Pharmacy borrowed money from the bank, what type of cost is this? A. Debit
B. Asset
C. Credit
A. number of people coming to hajj in the
A pharmacy in mecca wants to increase sales. The pharmacist was asked
following year.
topropose a business plan to improve sales. Which of the following is the
B. past sales, current situation, future
pharmacist required to research?
economy forecast/outlook
Study design
A doctor has a research question comparing the efficacy of two different
Random Control Trials (RCTs)
treatments, what research type is the most appropriate to answer this?
A doctor wants to compare between cefazolin and oxacillin in treating
Random Control Trials (RCTs)
infection, what source should he check?
Which of the following is an example of primary resource? Look for RCTs in PubMed
What is the suitable source of information for pediatric treatment? Harriet Lane Handbook
Which information source can be used to check for drug interactions? Lexicomp
Where to look for data on new clinical trials for management of diabetes? Pubmed
Which part of study report is the objectives? Introduction
Which type of study should be conducted to assess cause of rare disease
Case-control study
or diseases with long latency period?
The authors report two isolated gastrointestinal tuberculosis in renal
transplant recipients that illustrate the difficulty of making this diagnosis
and a brief review of the literature on its clinical presentation, diagnosis Case-series report
and therapeutic approach. What is the type of study in the above-
mentioned abstract?
A trial compared mortality in patients taking infliximab (7%) and patients
3%
taking old drug (10%). Calculate absolute risk reduction?
Calculate prevalence of disease if 500 patients have disease per 10,000
500/10,000*100 = 5%
people?
In which phase of clinical trials is the investigative drug compared with
Phase III
another drug?
Which of the following documents created by the world medical
association is the statement of ethical principles for medical research Declaration of Helsinki
involving human subjects?
A study located a group of subjects with lymphoma and went to identify
which subjects are smokers and which are not. Which of the following Cross-sectional study
study designs is used to conduct the study in the scenario?
What type of study is a self-controlled study? Crossover
A drug was discovered in year 2000, clinical trials started in 2005, and the
2020
drug was approved in 2010. When will the patent expire?
Systemic review is which type of resource? Secondary research
What is the concept, conclusion, or measurement can correspond
accurately to the real world. Also, to what extent the study measures what Validity
it is set to do?
what is a measure of the truth value of qualitative research, or whether
Credibility
the study findings are correct and accurate?
What is the consistency of the results (if we repeat experiment would we
Reliability
get same results)?
A patient has heart failure and stroke but does not have Atrial fibrillation.
Heart failure and stroke not Afib
How would you search for this in medical website?
Which of the following is at the bottom of the studies pyramid? Expert opinion/case studies
A. Gambling
In a study of quality of life, they asked people what remaining life they
B. Time trade off
would sacrifice in-order to get rid of a disease. What kind of measure is
C. Health utility index
this?
D. Willingness to pay
A. Micromedex
Which of the following is the most appropriate source for information B. Orange book
compounding medications? C. A drug information website
D. United states pharmacopeia (USP)
A. Endemic
Approximately 415 million people have been diagnosed with diabetes. B. Epidemic
Which of the following terms is referred to in the mentioned statement? C. Incidence
D. Prevalence
A. Endemic
Approximately 415 million people have been diagnosed with diabetes in
B. Epidemic
2022. Which of the following terms is referred to in the mentioned
C. Incidence
statement?
D. Prevalence
A. Case Series
Which of the following is the type of studies considers data from multiple B. Double-blind study
studies of different designs to draw conclusions? C. Systematic review
D. Consensus statement
A. Safety, Efficacy, and ADME
What does animal studies test for in pharmaceutical development of B. Potency
drug? C. Pharmacodynamics
D. Pharmacokinetics
Pharmacognosy
A colleague of yours asked you to identify the content of green leaves
herbal supplement, he's been using for the past two weeks (as tea drink)
to help him lose weight and boost his energy. He mentioned that he is
starting to have the following symptoms: tremors, palpitation and lack of Alkaloids
sleep (insomnia). You prepared aqueous and organic solutions from the
leaves and performed phytochemical screening tests to identify the plant
show (positive Dragendorff's reagent)
A patient has received an organ transplant took herbal treatment which
St John’s wort
caused organ failure, what is the likely herbal?
Gingko, Ginger, Garlic, Angelica, Anise, and
Which herbal drugs increase INR?
Cranberry juice
Green Tea, St. John’s wart, and Coenzyme
Which herbal drug decrease INR?
Q10.
Which herb increases blood pressure? Licorice
Which medication is used for amebiasis? Ipecac
Which herb has serotonin properties similier to ondansetron and used to
Ginger
treat emesis?
What is myrrh used for? Cough
Which is the least statin to interact with grapefruit juice? Pravastatin
Breastfeeding mother drinks 8 cups of coffee per day and takes anti-
hypertensive, cholesterol, and DM medications. Her baby has sleep Coffee
disturbance, what caused that?
A lactating mother is complaining of migraine. Her medication has
resulted in reduced milk production. Which medication was she Ergotamine
prescribed?
A 50-year-old man on warfarin therapy started drinking an herbal tea
A. Anise
extra (one gallon daily). A week later his INR dropped to 1.35 prior to that
B. Ginger
time, his INR had been in the range of 2.5-3. Once the herbal tea was
C. Green tea (contains vitamin K)
discontinued, the patient's INR came back up to 2.50. Which of the
D. Chamomile
following is the content of the herbal tea extract?
A. St. John’s Wort
Which herb causes hypoglycemia?
B. Fenugreek
Vitamins / Minerals
Which vitamin is given for breast cancer prophylaxis? Vitamin D
Which drug acts on vitamin K? Warfarin
What enhances the activity of insulin? Chromium
Which Vitamin D is used in infants? Cholecalciferol (Vitamin D3)
Which vitamin deficiency causes Beri-Beri? Vitamin B1 (thiamin)
Which vitamin is involved in the conversion of methionine to cysteine? Vitamin B6 (pyridoxine)
Which of the following is recommended after gastric sleeve? Vitamin B12
A patient with gastric bypass, which vitamin deficiency will he have? Vitamin B12 deficiency
A patient on metformin is experiencing tingling in fingers and toes. What
Vitamin B12
supplement should be given?
Infant with nasal bleed, what is the likely deficiency? Vitamin K
Vitamin C (ascorbic acid) facilitates iron
What is the role of vitamin C when given with iron?
absorption and creates an acidic
environment preventing oxidation of
ferrous iron to ferric acid.
TB is one of the most common infectious diseases prevalent in Saudi
Arabia, the main drugs used are isoniazid and pyrazinamide, which can Pyridoxine (Vitamin B6)
cause sideroblastic anemia. Which treatment should be given?
What is the active form of Vitamin D? Calcitriol
What is the carrier of vitamin b12? Intrinsic Factor
Which enzyme converts vitamin D to the active form (calcitriol)? 1-α hydroxylase
Which agent decreases warfarin effect? Vitamin K
A child swallowed rat poison (Rodenticide). What is the suitable
Vitamin K
treatment?
A. Vitamin A
B. Vitamin B
Which vitamin deficiency causes poor baby growth?
C. Vitamin C
D. Vitamin D
A. Take vitamin C before onset of
A patient wants to take vitamin C for infection to improve recovery, what
symptoms
advice should you give?
B. Take Vitamin C after 2 days of symptoms
A. Selenium
A patient underwent gastric sleeve and came after few days with fatigue
B. Vitamin B12
and couldn’t stand on her feet. Lab tests shows high levels of
C. Vitamin D
homocysteine. Which of the following is she deficient of?
D. Iron
A. Calcium
B. Phosphate
Vitamin D enhances absorption of which of the following?
C. Calcium and Phosphate
D. Iron
A. Mercury
Which type of heavy metal overexposure causes a blue discoloration to B. Lead
appear along the gum line of the mouth? C. Chromium
D. Copper
Genetics
What is the most common gene mutation? SNP
Which of the following pairs represent the purine nitrogenous bases? Guanine (G) and Adenine (A)
A patient is taking Irinotecan and has UGT1A1*28 mutation, what is this
Neutropenia
patient at risk of?
A patient is taking Phenytoin and has HLA-B 1502 mutation, what is this
Hypersensitivity reaction
patient at risk of?
In which genetic mutation is Imatinib used? Philadelphia-chromosome (BCR-ABL)
A patient with colorectal cancer has poor response to Cetuximab, what is Patient has mutated KRAS (Cetuximab only
the absolute neutrophil count ANC likely cause? effective on non-mutated KRAS wild type).
A patient is prescribed warfarin but has VKORC1 polymorphism, what is
Start on lower dose and monitor INR
the suitable intervention?
Type of DNA promoter sequence, which
What is the function of TATA box? specifies to other molecules where
transcription begins.
Drug A with CYP1A2 and another drug B is CYP2D6 inducer, what is the
No interaction
possible interaction between them?
What is the Risperidone pharmacogenomics? Need normal CYP2D6 for metabolism
What is the Tramadol pharmacogenomics Need normal CYP2D6 for metabolism
Which of the following polymorphic enzymes is responsible for conversion
CYP2D6
of codeine to morphine?
A patient was started on codeine 15 mg orally as needed for post-
operative pain. One day after starting codeine, the patient presents to ER
Rapid metabolizer of CYP2D6
with confusion, headache, and SOB. Which of the following most likely
described the patient’s genotype?
A child who was given codeine experiences dizziness and respiratory
The child is a Rapid metabolizer of CYP2D6
depression, what is the likely cause?
Patient with CYP2C19* poor metabolizer is prescribed clopidogrel, what is
Reduced effectiveness of clopidogrel
the likely impact?
Increases risk of neutropenia especially if
What are the implications UG1TA1*28 in patient prescribed Irinotecan?
two copies (Homozygous)
Omeprazole reduces effectiveness of
clopidogrel by inhibiting CYP2C19, which is
What is the interaction between omeprazole and clopidogrel?
required for activation of clopidogrel to
active form.
Which of the following drugs requires HLA-B 1502 to be checked before
Carbamazepine
use?
Case of patient that has been taking simvastatin 80 mg he has SLCO1B1 Hold statin for 2-4 weeks and re-start on
521C allele – patient has signs of myopathy and elevated CPK, what lower dose or a different statin (depends
should be done? on answer choices)
Which one is a non-synonymous single nucleotide polymorphism? Nonsense SNP
Which enzyme is responsible for the oxidation of phenytoin? CYP2C9
A patient with breast cancer is CYP2D6 poor metabolizer, which
Tamoxifen
medication she will not benefit from?
Which pharmacogenomic testing is required with Rituximab? CD19
Which mutation may occur in colon cancer? KRAS mutation
Which gene mutation increases risk of thrombosis with oral
Prothrombin
contraceptives?
Patient with UGT1A1 activity, which drug will this increases risk of side-
Atazanavir
effects of?
A-Raloxifene – only for prevention of breast
cancer
A post-menopausal patient with breast cancer is prescribed estrogen
B-Tamoxifen – metabolized to active by
therapy. She is a poor metabolizer of CYP2D6. Which medication will
CYP2D6
reduce progression-free survival?
C-Anastrozole – 1st line in postmenopausal
women
A- CYP2C9
B- CYP3A4
Mutation in which enzyme would affect metabolism of Atorvastatin?
C- CYP2D6
D- CYPC19
A. Synonymous – nonsense
B. Non-synonymous- nonsense
Which of the following SNP does not change coding of protein?
C. Synonymous – Silent
D. Non-synonymous – missense
A – Nonsynonymous - Missense
B – Nonsynonymous - Nonsense
Which single nucleotide polymorphism (SNP) causes abrupt stop codon?
C – Synonymous - Missense
D – Synonymous - Nonsense
A. Carbamazepine
Patient with epilepsy is HLA-1502 positive, what is the safest anti- B. Lamotrigine
epileptic? C. Fosphenytoin
D. Levetiracetam
Pregnancy
Which drug is contraindicated in pregnant women? Tretinoin (Acretin)
A female is taking isotretinoin and wants to become pregnant. What Stop isotretinoin 1 month before getting
advice should you give? pregnant.
Female patient on Finasteride and wants to conceive, what counselling Inform her that she cannot become
should be given? pregnant while taking finasteride.
What is the most appropriate anti-convulsant for a pregnant woman? Lamotrigine
Which is the suitable mood stabilizer for a pregnant woman with bipolar
Lamotrigine
disorder?
Woman taking levothyroxine has become pregnant. What is the most
Inform prescriber to increase dose
appropriate course of action?
Which antibiotic is the safest for pregnant woman with UTI? Cephalexin
A patient on 100 mcg of Levothyroxine becomes pregnant, what is the
125 mcg (25% increase)
appropriate dose to give?
Which vaccines are contraindicated in pregnant patient? Live vaccines
Pregnant patient with hyperthyroidism, what is the recommended PTU for 1st trimester then Methimazole for
treatment? 2nd and 3rd trimester.
What is the suitable treatment for a pregnant woman with hypertension? Labetalol
Pregnant with cat, what is she at risk of? Toxoplasmosis gondii infection
Which vaccine can pregnant not take? MMR
Which of the following vaccines is contraindicated in pregnant women? BCG vaccine
A woman is planning to conceive and is taking levothyroxine. What should
Adjust dose during 1st trimester.
you do?
When should mycophenolate be stopped in pregnancy? First trimester
A- Continue on Valproic acid
B- Stop Valproic acid
A female patient with epilepsy controlled on valproic acid wants to
C- Switch to Phenytoin
conceive, what is the best action?
D- Slowly taper down dose and slowly
increase dose of Levetiracetam
Pathology
high blood pressure (other: gout,
What are the health complications of polycythemia (high RBC conc.)?
headache, blood clots).
What are the symptoms of severe malaria infection? Bleeding
Low magnesium, potassium, and
What electrolytes are low in refeeding syndrome?
phosphate
What is low creatinine clearance a measure of? Renal failure
Which lab results show hypothyroidism? Low T4 and high TSH
Moderate Nightmare Disorder (permanent
Patient is experiencing 2-4 nightmares per week; what type of nightmare
is daily, and mild is less
disorder is this?
than 1 per week)
What type of anemia is caused by iron deficiency? Hypochromic microcytic anemia
A patient receiving TPN has hypomagnesemia, hypophosphatasemia, and
Refeeding Syndrome
hypokalemia, what is this condition called?
Physical examination in sepsis? ??
What is the physical exam for peripheral oedema? Pitting test
A patient comes in with signs of meningitis including a headache. The
doctor lifts the patient’s neck to check if knee flexion occurs. What type of Brudzinski sign
examination is this?
Healthy male happily married for 25 years now has erectile dysfunction
wants treatment and reluctant to use medicine or injections, which is the Vacuum erection devices
most suitable management?
Used as indicator of nutritional status in
What does mid arm circumference measure?
children.
What is the purpose of the skin fold test? Calculate fat percentage
How to check for respiratory infection? Auscultation
A cardiac patient with edema, how can you check heart rate? Auscultation
What advice would you give to patient who is traveling and is scared of Antibiotic prophylaxis (start 1-2 days before
getting malaria? travel and up to 4 weeks after travel)
A doctor has contact with patient who has c. difficle, what is the
Use water and soap
appropriate action?
What is the end product of purine metabolism? Uric acid
Asthmatic patient uses Ventolin 3 days a week, wakes up 3 times at night,
and asthma interferes with physical activity. What is the asthma severity moderate
category?
A patient he want go to Africa and afraid of travel diarrhea what is the
Don’t eat raw meat or fish
recommended ?
Broad view that attributes disease
Which of the following best described the “biopsychosocial approach of outcomes to the variable interaction of
health”? biological factors, physiological factors, and
social factors.
Undercooked meat, unpasteurized milk,
Patient is infected with Brucella, what is the likely source?
and direct contact with infected animals.
What will occur if there is deficiency in phylloquinone? Bleeding
What is the condition in which the body's immune system fails to react to
ANERGY
an antigen?
Which measure is used for COPD? mMRC
Which examination is used for neuropathy in diabetes? Monofilament test
A patient is a smoker and is taking NSAID, How many risk factors for
Smoking and NSAID use
developing ulcers does the patient have?
Which is given to patient to measure glomerular filtration rate (GFR)? Insulin
Lumps or swelling in the breast or
underarms, change in skin color or texture
What are the early signs of breast cancer?
of breasts, pain in breast, dimpling of
breast skin.
What is an important information that must be asked to patient with
Travel History
malaria?
A. Frontal Lobe
Which part of the brain is affected in patient who had an accident which B. Parietal Lobe
resulted in mental retardation and speech difficulties? C. Amygdala
D. Occipital
A- Mild
What is the asthma severity in patient who experiences daily symptoms B- Moderate
and 2-3 times night awakening and uses inhaler several times per day? C- Persistent Asthma
D- Intermittent
A- Mild
What is the asthma severity in patient who experiences symptoms 2 days B- Moderate
per week, with no exercise limitation and FEV1 >85%? C- Persistent Asthma
D- Intermittent
A 40-year-old patient case and asks what examination should patient A- Get checkup for lung cancer
receive? B- Get checkup for diabetes
C- Wait until after age 45 to get diabetes
checkup
D- Wait until after age 45 to get lung
cancer checkup
A. Need to use hand sanitizer.
A doctor was in contact with patient with C. difficile, The doctor wiped his B. Antiseptic wipe is enough
hand with antiseptic wipes ? C. Need to wash hands with soap and
water.
A. continuous feeding regimens
B. high gastric residual prior to feeding
Which of the following increases the risk of aspiration during enteral
C. feeding with high-carbohydrate content
feeding administration?
D. feeding a patient while in the upright
position
A. period between infection and
appearance of symptoms.
What is the definition of prodrome?
B. period with non-specific symptoms.
C. period after recovery from illness.
Man is at risk of developing Type 2 diabetes and has family history of type A – Insulin
2 diabetes. What should be recommended to patient to prevent him from B – Sulfonylureas
developing type 2 diabetes? C – Aerobic Exercise
Pharmacology
Which anticancer drugs target the mitosis stage? Vina Alkaloids and Taxanes
Increase osmotic pressure (osmotic
What is the mode of action of mannitol? diuretic used to reduce intracranial
pressure)
What is the mode of action of topiramate? Sodium channel blocker
What is the mode of action of atropine? Anti-muscarinic agent
What is the mode of action of bisoprolol Beta 1 selective antagonist
What is the mode of action of Prazosin? Post-synaptic Alpha-1 inhibitor
What is the mode of action of Zafirlukast? Leukotriene receptor antagonist
Inhibitor of thyroid peroxidase thus
What is the mode of action of Methimazole? inhibiting incorporation of iodine and
production of thyroid hormone.
Methyldopa is converted to methyl
What is the mode of action of Methyldopa? norepinephrine centrally and acts as an
alpha-2 agonist
What is the mode of action of Fluorouracil? Thymidylate synthase inhibitor
Alkylating agent, which causes cross-links
What is the mode of action of cyclophosphamide?
between DNA strands.
What is the mode of action of clonidine? Alpha-2 receptor agonist
What is the mode of action of Ciprofloxacin? DNA gyrase and Topoisomerase IV inhibitor
What type of beta-blocker is propranolol? Non-selective beta-1 and beta-2 inhibitor
Which one is a long-acting insulin? Insulin Glargine, Insulin Degludec
What happens if you activate alpha 1 receptor? Vasoconstriction which increases BP
Which cephalosporin class does ceftazidime belong to? Class 3 Cephalosporin
Which drug targets dihydrofolate reductase inhibitor? Methotrexate
Which anti-diabetic class does sitagliptin belong to? DDP-4 D
Which class of antidiabetic Empagliflozin belongs to? SGLT-2 inhibitor
What is the class of Isoproterenol? Beta-1 and beta-2 receptor agonist
Which of the following anti-arrhythmic drugs can be considered as a rate
Amiodarone
control drug?
What is the clinical use of acetylcysteine? Mucolytic
A patient is experiencing cough and mucus. What medication can he use? Guaifenesin
What is the treatment of ductus closure? Indomethacin
What is the treatment of Meniere’s disease (feeling dizzy, vertigo,
Betahistine
tinnitus)?
What is the treatment choice for Tape worm infection? Praziquantel
What is the suitable treatment to reduce acute hypertension in patient
Nicardipine
with stroke?
A patient has Duchenne muscular dystrophy, which steroid medication is
Deflazacort
used to treat this?
Which antihistamine is also a weak serotonin reuptake inhibitor? Tripelennamine
Treatment choice for ulcerative colitis? Sulfasalazine
Which corticosteroid is applied topically? Clobetasol propionate
Which drug is decreased lower esophageal sphincter tone? Amlodipine
A patient with peritonitis that is positive for Staphylococcus aureus and
vancomycin
Enterococcus faecalis. What antibiotic should be given?
What is the first line treatment for patient with severe Alzheimer’s
Donepezil
disease?
A patient has palpitations, tremor, and weight loss, what is the best
Methimazole
choice?
What is the treatment of pin worms? Albendazole or Mebendazole
What is the safest treatment for pregnant patient with UTI? Amoxicillin
1st line = Stimulants (e.g., senna or
Which laxatives should be given to patient with opioid-induced
bisacodyl) +/- stool softener (e.g.,
constipation due to morphine?
docusate)
Which biosimilar drug is approved for the treatment of moderate to
Omalizumab
severe allergic asthma?
Which condition requires patient to take Epoetin? Renal disease
Which one is the least sedating antipsychotic? Risperidone
Which anti-cancer drug works by inhibit formation of microtubules by
Vincristine
binding to tubulin?
Which anti-cancer drug works by stabilizing microtubules thereby
Paclitaxel
preventing cell division during M phase?
Gravis disease treatment? Methimazole
An antibiotic that inhibits the ribosomal subunit 50S inhibits which
Elongation
process of translation?
An antibiotic that inhibits the ribosomal subunit 30S inhibits which
Initiation
process of translation?
What does the QRS on an ECG represent? Ventricular depolarization
What does the QT interval on ECG represent? Ventricular action potential
L-O-T (Lorazepam, Oxazepam,
What choice of benzodiazepines should be used in elderly patient?
Temazepam).
Which anti-depressant should be used in obese 18-year-old girl? Bupropion
Diazepam, Clonazepam, Flurazepam,
Which of the following is a long-acting benzodiazepine?
Quazepam
Scientists are conducting clinical trials assessing the use of anti-psychotics Risk of extra-pyramidal symptoms and
in depression, what is the likely concerns? metabolic problems.
Which of the following drugs require extra precautions during handling to
Methotrexate
ensure medication safety?
Which of the following may decreases heart rate? Metoprolol
A patient on dialysis was prescribed sodium polystyrene, what is the
To reduce potassium levels
reason?
A patient with reduced kidney function is prescribed gentamicin and
Both
ampicillin, which ones require renal adjustment?
It is the precursor of glutathione
Why use of cysteine in chemotherapeutic agent? contributive to oxidative stress control and
stimulates cellular bioenergetics.
How does Mifepristone cause abortion? Mifepristone is an anti-progestogen
Take on an empty stomach with a full glass
What advice should be given to a patient taking alendronate?
of water while sitting upright
Given once an hour or as continuous IV
Aminocaproic acid is used to stop bleeding, how should the medication be
infusion for 8 hours or until bleeding is
given?
controlled.
What is the advantage of IV administration? Bypass first-pass metabolism
What is the first choice of treatment for a patient experiencing
IV fluids
hypotension?
A patient has albuminuria >300mg/mL, what is the suitable treatment? Lisinopril
What is the cause of cough with Lisinopril? Bradykinin
What is the washout period for Fluoxetine? 5 weeks
Methimazole(withdraw amiodarone if
What is the treatment of thyroid toxicity?
possible).
Which drug is an IV direct thrombin inhibitor? Argatroban/Bivalirudin
Valacyclovir has higher bioavailability and
Which one has higher oral bioavailability, Valacyclovir or Acyclovir?
longer duration of action
How does digoxin work in the heart? Increase cardiac contraction
Which benzodiazepine is used in general anxiety disorder (GAD)? Diazepam, alprazolam, lorazepam
What requires monitoring in patient receiving spironolactone? Potassium levels
Monitor ANC every week for 6 months
then every 2 weeks for 6 months then
What monitoring is required with Clozapine?
monthly. Stop therapy when ANC
<1,000/m3.
A patient is prescribed Rituximab, what monitoring should be conducted
Hepatic Panel
before initiating treatment?
Which parenteral anticoagulant requires routine monitoring of
Intravenous Unfractioned Heparin
coagulation lab parameters?
Which medication is added to levodopa to reduce the required dose of
Carbidopa
levodopa?
A patient with hypertension has erectile dysfunction. What is the
Sildenafil
recommended treatment?
A patient with cholesterol level of 195 mg/dl, what is the appropriate
Atorvastatin 40 mg
medication?
What is the treatment choice for group B streptococcus infection? Cephalosporin or penicillin.
What is the treatment choice for group B streptococcus infection in
Vancomycin
patient with penicillin allergy?
Trimethoprim/sulfonamide dosing is based on which one? Trimethoprim
A patient has symptoms of hyperthyroidism (weight loss, insomnia,
increased sweating etc.) and labs show high T4 levels and low TSH levels, Hyperthyroidism
what is the likely diagnosis?
severe lower respiratory infection with
underlying comprising condition
When inhaled ribavirin is given?
(premature neonates, cardiopulmonary
disease, or immunosuppression).
Premature infants (<29 weeks gestation)
and children <12 months with respiratory
When IM Palivizumab is given?
or heart conditions as monthly prophylaxis
during fall/winter season.
What is the drug of choice to treat patient with Anaphylaxis? Epinephrine
What is the treatment choice for MSSA infection? Nafcillin or Oxacillin
Which drug should be avoided in patient with arrhythmia, which drug
Azithromycin
should be avoided?
A patient is experiencing symptoms of allergy. The patient works as a truck
driver. He comes to the pharmacy asking for medicine to treat his allergy. Desloratadine (non-sedating antihistamine)
What is the suitable treatment?
A patient had a car accident after taking medication. What is the likely
Diphenhydramine (sedative)
medication that caused this?
The Naranjo score of reported ADR is 8, what does this mean? Probable
What does Naranjo score of 3 mean? Possible
What should be monitored in patient taking Fingolimod? ECG
A patient with sneezing, cold, and runny nose. Which treatment should
Pseudoephedrine
you give?
Patient suffering of asthma and hypertension, which medication should
Propranolol
they avoid?
Child with bowel, leg, and bone pain, what should be given? Ergocholecalciferol
- Beta-lactam
- Glycopeptides - Macrolides
Which antibiotic is time-dependent?
- Tetracycline
- Vancomycin
- Fluoroquinolone
- Aminoglycosides
Which antibiotic is dose-dependent?
- Metronidazole
- Amphotericin B
10 mg/min or 500 mg/hr (NMT 5 mg/mL =
Maximum infusion rate of vancomycin?
2 mL/min)
What is the maximum statin dose for patient taking verapamil? 10 mg
A patient who is Hajj visitor is prescribed Diazepam as needed for
15 mg
insomnia, what is the maximum dose?
What is the appropriate paracetamol dose for a child weighing 20 kg? 10-15 mg/kg Q4-6
patient is diagnosed with pulmonary embolism, patient weight is 120 kg
120 mg BID (1 mg/kg BID based on TBW).
and his is 1.72 m (BMI is 40), what is the appropriate dose?
0.4 – 2 mg IV/SC/IM/Inhalation if non-
opioid dependent.
0.1-0.2 mg if opioid-dependent (to avoid
What is the correct dose of Naloxone?
acute withdrawal)
Repeat every 2-3 mins if needed (max. 10
mg).
0.01 mg/kg IV once; may repeat with 0.1
What is the dose of naloxone for children?
mg/kg
Adult: 0.5-2 g/day PO divided q6-8hr for 7-
Clinical case asking for vancomycin dose and interval for administration? 10 days
Pediatric: 10-15 mg/kg every 12 hours.
Stroke prophylaxis: 5 mg BID (2.5 mg if age
>80 or weight <60 kg). DVT/PE prophylaxis:
What is the correct dose of Apixaban? 2.5 mg BID.
DVT/PE treatment: 10 mg BID for 10 days
then 5 mg BID.
15 mg BID for 21 days then 20 mg OD
What is the correct dose of Rivaroxaban?
(avoid if CrCl <30).
VTE prophylaxis: 5,000 units SC Q8-12 hrs
VTE treatment: 80 mg/kg IV bolus (or 18
What is the correct dose of Heparin? units/kg/hr if infusion)aACS/STEMI
treatment: 60 units/kg IV bolus (or 12
units/kg/hr if infusion)
When should trough levels be measured for vancomycin? 30 mins before 4th dose (after 3rd dose).
What is the dose of ceftriaxone for meningitis treatment? Ceftriaxone 2g IV BID for 7-14 days
A. Hyperphosphatemia
Case of patient prescribed Cinacalcet and Sevelamer. What is this given to B. Hypophosphatemia
treat? C. Hypercalcemia
D. Hypocalcemia
A 28-year-old female with severe pain during menses and intercourse and A. Oral contraceptive pill.
an ultrasound shows cyst in the right ovary. What is the first line B. Estrogen
treatment? C. Laparoscopic insertion
A. Rifampin
Which drug affects levels of oral contraceptives? B. Amoxicillin
C. Ciprofloxacin
A- Blockage of post-synaptic alpha-1
receptors
B- Blockage of pre-synaptic alpha-1
receptors
What is the mode of action of Doxazocin?
C- Blockage of post-synaptic alpha-2
receptors
D- Blockage of pre-synaptic alpha-2
receptors
A- Glibenclamide
B- Metformin
Which drug is considered an insulin secretagogue?
C- Pioglitazone
D- Dapagliflozin
A. Adrenaline
Patient has an anaphylactic shock, and he was given medication that is
B. Phenyl ephedrine
inotropic and chronotropic, which medication was he given?
C. Isoprenaline
A. class IV
Which of the following is the Vaughan Williams Classification for the B. class lll
antiarrhythmic drug procainamide? C. class II
D. class IA
A. It can be taken with calcium
supplements or dietary foods with no
problems
B. Take it in the morning on an empty
stomach, at least 30 mins (before
Which of the following counselling points is best to discuss with a patient
breakfast)
who is taking levothyroxine?
C. If you miss a dose, make sure to double
the next scheduled levothyroxine dose
D. Take it immediately after the breakfast
with a large glass of water to avoid
stomach upset
A. Decreases excretion of cholesterol
B. Lowers high density lipoprotein (HDL)
level
Which of the following best describe the main pharmacological property
C. Increases low density lipoprotein (LDL)
of cholestyramine?
level
D. Increase the efficiency of lipoprotein
removal
A. Prednisone
B. Triamcinolone
Which of the following glucocorticoids has the lowest potency?
C. Hydrocortisone
D. Dexamethasone
A. Increasing the glomerular filtration
Which of the following is the mechanism by which probenecid increases B. Increasing the intestinal absorption
the duration of action of ampicillin when administered at the same time? C. Inhibition of renal tubular secretion
D. Inhibition of renal tubular reabsorption
A. it is a vasodilator
B. it is a potent vasoconstrictor
Which property of nicotine is attributed to abortions?
C. it reduces uterine blood flow
D. it reduces vascular uterine resistance
A. Oxazepam
B. Diazepam
Elderly patient suffers from insomnia, BDZ of choice?
C. Lorazepam
D. Temazepam
A. Dextromethorphan
B. Diphenhydramine
3 years girl with dry cough what is recommended?
C. Honey
D. Vicks
A-Diltiazem
Which of the following drugs cause insomnia and should be taken in the
B- Prednisone
morning?
C-Salbutamol as needed
Which of the following medications has the same mode of action as A. Cephalexin
penicillin? B. Neomycin
C. Tetracycline
A. inhibit intracellular release of Ca2+ from
sarcoplasmic reticulum.
What is the mode of action of Dantrolene?
B. inhibit intracellular release of Na+ from
sarcoplasmic reticulum
Patient have angina and he suffered from low BP and have syncope, which A- Hydralazine
antianginal agent may be associated? B- Isosorbide Mononitrate
A. Estrogens that undergo CYP450
A patient was given oral estrogen therapy. When she read the package, metabolism to inactive compound.
she noticed that it says that the medication contains conjugated estrogens B. Glucorinated estrogens, which makes
(Equilin and Estrone). When she googled the names of these estrogens, them active.
she found the following structures. What are conjugated estrogens? C. Estrogens that undergo phase II
conjugation to inactive form
A. Fluconazole
What is the treatment of esophageal candidiasis? B. Voriconazole
C. Amphotericin B
A. 5% dextrose
Which of the following is a high alert drug? B. 0.9% NaCl
C. 50% dextrose
A. Verapamil
Which one of the following is a rate control drug used in Atrial Fibrillation? B. Digoxin
C. Amiodarone
A. Decrease heart contractility.
What is the mechanism of action of cardiac myosin inhibitors
B. Treat hypertension.
(Mavacamten)?
C. Decrease oxygen demand.
A. Metoprolol
Woman with a disorder causing discoloration of her fingers (Raynaud’s B. Nifedipine
disease). What is the best treatment for Raynaud’s? C. Methyldopa
D. Hydralazine
A. Methotrexate
Which of the following chemotherapy is preferred in patient with lung B. Vinca alkaloids
cancer? C. Nitrosourea
D. Taxane
Clinicals
An elderly patient with diabetes has a HbA1c level of 12% and is on 4 anti-
Reassure patient that insulin is needed due
diabetic medications, one of which is insulin Patient refuses to take insulin
to high A1c and give advice to control
due to fear of weight gain. What is the most appropriate intervention? weight.
A patient on insulin refuses to take insulin due to fear of weight gain, what
Try to explain to patient the benefits of
is the best course of action? treatment to change their belief.
Explain benefits and risk of not receiving
A woman with cancer refuses to receive chemotherapy or surgery. What is
treatment so she can make an informed
the best course of action?
decision.
Starch (amylase produced by pancreas
A patient with damaged pancreas consumes a high carb meal. The
catalyzed digestion of starch into maltose
digestion of which type of carbohydrates will be most affected?
and glucose)
A doctor has increased the dose of a patient’s anticonvulsant. What is the
Zero seizures
treatment target?
What question should be asked to patient on warfarin? Do you have gum bleeding?
Patient comes to pharmacy with pinpoint pupils, what is the likely cause? Heroin Overdose
A female with hypothyroidism has developed osteoporosis, what is the Alendronate (High doses of levothyroxine
appropriate treatment? can cause bone resorption)
What is the first-line treatment for patient with osteoarthritis where non-
Topical NSAID (gel or patch).
pharmacological measures were ineffective?
Patient with hypothyroidism, which deficiency is he likely to have? Iodine deficiency
Patient with E. coli UTI and given TMP/SMX but it turns out to be resistant.
Ciprofloxacin
What is the appropriate alternative?
Cefotetan, Cefoxitin, ampicillin/sulbactam,
A patient is at risk of post-operative gram-negative bacterial infection,
Ertapenem, or Metronidazole + (Cefazolin
which antibiotic should be used?
or ceftriaxone).
A 60 yo patient presents with meningitis, what is the appropriate
Ceftriaxone + Vancomycin + Ampicillin
treatment?
A neonate patient presents with meningitis, what is the appropriate
Ampicillin + Gentamicin
treatment?
Patient vomits two hours after taking levoynorgestrel, what is the If more than two hours, no action. If within
appropriate course of action? two hours, requires repeat dose.
Patient with multiple sclerosis experiences multiple relapses, what is the
Natalizumab
appropriate treatment?
What is the best treatment for a boy with motor hyperactivity? methylphenidate
Which anti-hypertensive drug that can be used in patient with bilateral
Amlodipine
renal stenosis?
A patient is taking amlodipine and his BP is still uncontrolled, what is the
Losartan
appropriate drug that can be added?
What is the suitable oral treatment for patient with multiple sclerosis who
Fingolimod
is experiencing 2 relapse per year?
Patient going into surgery, which medicines should be discontinued? Anti-coagulants
A patient who has heart failure and Diabetes mellitus. What is the
Furosemide
recommened for pulmonary edema?
Which should be initiated in kidney failure patient with ankle edema and
Furosemide
heart failure?
A patient was prescribed both frusemide and hydrochlorothiazide, why is Patient has severe heart failure that is
this? unresponsive to one diuretic
Patient still suffering from UTI even after adherence to antibacterial
Change to another antibiotic
treatment, what is the best course of action?
Use bronchodilator (SABA) +
Patient with Hospital-acquired pneumonia experiences bronchospasm?
Corticosteroid.
A patient is taking an antihypertensive drug, but it is still not controlled, Answer depends on age, race, and co-
what is the best course of action? morbidities
A patient taking 850 mg of metformin twice daily but still has uncontrolled
Increase dose of metformin
diabetes, what is the best intervention?
A patient with type 2 diabetes and HbA1C 13% on glargine 22 IU and
aspart 7 before each meal, blood glucose in the morning 118 and bedtime Increase aspart before dinner
280 what is the recommended?
A patient with altered kidney function is taking gentamicin and ampicillin, Both antibiotics require renal dose
what is required? adjustments
A patient is taking the maximum statin dose and cholesterol levels are still
Add Ezetimibe
not on target, what is appropriate intervention?
Digoxin Immune Fab (DigiFab) – each 40
A patient is experiencing digoxin toxicity, what treatment should patient
mg vial binds 0.5 mg digoxin (max. 20
receive?
vials).
Do not administer Digifab (Digoxin level
A patient’s digoxin levels were obtained 2 hours after ingestion of digoxin
needs to be obtained at least 6 hours after
and digoxin level was 2.7. What is the best course of action?
digoxin dose).
A nurse was infected with Hepatitis B from infected patient blood, what is Hepatitis B Vaccine + Hepatitis B
the appropriate treatment? immunoglobulins
A patient comes to the pharmacy for emergency contraceptives (Plan B – Must be taken within 72 hrs of
Levonorgestrel), what advice should you give her? unprotected sexual intercourse.
A patient case with hyperkalemia above 6 and hyponatremia, what’s the Urgent treatment of hyperkalemia first
first step treatment and duration? (according to MOH).
A patient who came to receive vaccination was found to have high blood
Refer patient to physician
pressure in two readings, what is the best course of action?
A hypertensive patient with BP 177/97 presents to your pharmacy, what
Refer patient to physician
should you do?
A patient comes with pain in his toe and it’s bleeding, what advice should
Refer patient to physician
you give patient?
A patient was taking non-pharmacological treatment for rheumatoid
Methotrexate
arthritis but has not helped. What treatment should he be given?
What is the appropriate first-line treatment for patient with severe
Donepezil
Alzheimer disease?
Patient using Rivastigmine tab for alzheimer’s, however he is not tolerant
Donepezil
with it, Which of the following is the best alternative drug?
A patient has hypoalbuminemia, what effect will this have on phenytoin Increase free serum concentration of
levels? phenytoin
A patient is taking Azathioprine 6-mercaptopurine has reduced levels of
Bone marrow suppression
Thiopurine methyltransferase (TPMT), what is this patient at risk of?
A patient with diabetes and CVD, what is the suitable treatment? Canagliflozin
Patient on Metformin, Sitagliptin and Pioglitazone recently diagnosed with
Pioglitazone
Hepatitis C virus, which drug should he stop?
Patient on Lisinopril, Amlodipine, Metformin and another drug the patient
will perform iodinated contrast which one need to be restarted after Metformin
48hrs of the on contrast?
A girl accidently took 2 doses of lamotrigine, what will she likely
Dose-dependent serious skin reactions
experience?
Patient with mechanical heart valve is taking warfarin and his INR is 2.5,
Keep the dose as it is
what dose adjustment should be done?
A patient on warfarin has INR of 11 and is experiencing minor bleeding.
Stop warfarin and give slow IV vitamin K.
What is the suitable intervention?
A patient with INR of 6.1, what adjustment should be made? Hold for 1-2 doses
Patient with an acute asthma attack was given salbutamol but has not
Oral Corticosteroids
improved, which treatment should be added?
A patient with rheumatoid arthritis is on methotrexate and leflunomide,
Infliximab (anti-TNFa antibody treatment)
what treatment should be added?
A 36-year-old man not known to have any medical illness diagnosed
Isoniazid + Rifampin + Ethambutol +
recently with an active pulmonary tuberculosis. Which of the following is
Pyrazinamide
the best initial treatment?
Anti-pseudomonal cephalosporins
A pregnant woman has pseudomonas infection. What is the suitable
(ceftazidime, cefepime) or
antibiotic option?
Piperacillin/tazobactam.
A patient has drug eluting states, which antiplatelet therapy should be so Dual anti-platelet therapy (e.g., Clopidogrel
prescribed? + Aspirin) for at least 6 months
Which laxative should be used in an elderly patient experiencing severe 1st line – bulk forming agent, 2nd line –
constipation? stimulant laxative
A patient with renal failure has low calcium levels, what is the appropriate
Calcitriol (1,25 dihydrocolecalcetrol)
calcium supplement to give?
Severe Persistent Asthma (step 4: treat
An asthmatic patient experiences more than 3 asthmatic attacks per day,
with SABA + medium-dose ICS-LABA
what is the severity classification?
inhaler)
Moderate Persistent Asthma (step 3: SABA
Asthmatic patient with FEV 75%, what is the recommended treatment?
+ low-dose ICS-LABA inhaler)
Asthmatic patient with 3 attacks per week, what is the severity Mild Persistent Asthma (step 2: SABA + low
classification? dose ICS)
Intermittent Asthma (step 1: rescue inhaler
Asthmatic patient with 2 attacks per week, what is the severity
only – low-dose ICS-formoterol or SABA +
classification?
low-dose ICS)
A 70 yo woman has dyspepsia and is experiencing pain in her bones and
acetaminophen was not effective for treating pain. What treatment Celecoxib
should be given?
A patient has experienced agranulocytosis after taking clozapine, what is Discontinue Clozapine, Restart once ANC
the suitable intervention? >1,500/mm3.
Supportive treatment (oxygen, IV fluids,
A patient is diagnosed with RSV, what is the suitable treatment?
etc.)
A 6-year-old boy is brought with complaint of uncontrollable spasm-like
Haloperidol (anti- psychotic is 1st line
movements and excessive blinking for four months. Chronic motor
treatment of tics)
disorder was diagnosed. Which of the following is the best management?
Score of 6
Calculate the CHA2DS2VASc score for 70-year-old male patient with HF
(CHF = 1, HTN=1, DM=1, Stroke/TIA=2, Age
who has hypertension, diabetes, and history of TIA?
65-74=1)
Calculate the CHA2DS2VASc score for 60-year-old female with diabetes
Score of 3, (HTN=1, DM=1,Sex(F)=1)
and hypertension?
A patient received Vancomycin 1g q12h for 3 days but there was not
Increase dose of vancomycin (therapeutic
improvement. The trough level was 8 mg/L. What is the recommended
level is 10-20 mg/L)
action?
Patient with hyperkalemia (6.1) already given Calcium gluconate, what
Insulin/dextrose
should you give next?
Bronchodilator (e.g., Albuterol or
What is the suitable treatment for exacerbations in COPD? Salbutamol) + oral/iv steroid + antibiotic
(azithromycin)
A patient is experiencing runny nose, cough, sneezing, and teary eyes.
Phenylephedrine
Which treatment provides fastest relief?
A patient with hypotension with hyponatremia, what is the appropriate
6-12 mEq/L in 24 hours
treatment?
A patient’s ECG show that patient has QT prolongation, T-wave elevation,
Hyperkalemia
and PR prolongation, what is the likely reason?
How can you differentiate between different types of Myocardial
Troponin and ECG
Infarction?
A patient is on spironolactone and captopril, what can this cause? Hyperkalemia and hyponatremia
Patient with potassium level 6.1, what is the appropriate
Calcium gluconate.
recommendation?
A T2DM patient was well managed with oral medication but with time
developed insulin insufficiency and his HbA1c is 8%, what is the suitable Long-acting Insulin
treatment?
A patient with diabetes has high post-prandial blood glucose but fasting Liraglutide (if we increase long-acting
glucose in range. He is taking liraglutide, metformin, and long-acting insulin it will increase risk of morning
insulin. Which one should be adjusted? hypoglycemia)
A. administer N-acetylcysteine.
A patient admitted to ER admitting suicide with acetaminophen 8 hrs ago, B. repeat lab test and observe for another
serum levels where 100 mcg/ml toxicity. She has nausea and vomited 2 hrs
several times in the last hour. What is the appropriate action? C. administer activated charcoal.
D. gastric lavage
A 62-year-old man is at risk of Type 2 Diabetes and his father has Type 2 A- Reduce Body Weight
Diabetes. He exercises occasionally. How can you decrease the risk? B-Initiate insulin
C-Increase exercise to 2-3 times per week.
(correct)
D- Initiate metformin
A-Fondaparinux 2.5 mg during
hospitalization
Patient did colorectal surgery and anti-coagulation is required (patient has
B-Enoxaparin 40 mg OD for 4 weeks
no contraindications). What is the suitable treatment?
C-Enoxaparin 40 mg OD during
D-Hospitalization Rivaroxaban
A-Give lower dose of Codeine
Cancer patient with pain. The doctor wants to start codeine treatment, B-Give higher dose of Codeine
but patient has 2 copies of CYP2D6 poor metabolizer. What is the suitable C-Give alternative as Codeine causes side-
treatment? effects.
D-Give alternative as Codeine ineffective.
A – Paracetamol
A child with fever and cough caused by virus. What is the suitable
B – Cefuroxime
treatment?
C – Oseltamivir
A- Stop calcium carbonate. (Patient needs
this for osteoporosis)
B- Separate amlodipine and calcium
carbonate 6 hrs before and after. (No
because type of interaction is
A patient with hypertension, osteoporosis and diabetes is taking
pharmacodynamic interaction not
amlodipine, calcium carbonate, metformin, and captopril. The pharmacist
pharmacokinetic interaction).
found a drug- interaction. What is the best action?
C- Increase dose of amlodipine. (this could
be the answer if BP increased)
D- No change in treatment required
(considered a minor interaction, which
requires close monitoring of BP)
A- Fibrinogen
B- CRP
A patient case with gastric cancer, which visceral protein parameter C- Albumin
should be measured? D- Hemoglobin
(Low albumin levels indicate poor
prognosis in gastric cancer)
Patient with lymphoma finished 2nd cycle of chemotherapy with
A- Prednisolone
cyclophosphamide, which drug will be used to lower the toxicity
B- Mesna
associated with us?
A- Estrogen
A patient with hysterectomy and develops vaginal dryness and hot flushes,
B- Estrogen and progesterone
what treatment should be given?
C- Lubricant
A-Aspirin and Clopidogrel
A patient requires dual anti-platelet therapy and has CYP2C19, what is the
B-Aspirin and Ticlopidine
suitable combination?
C-Ticlopidine and naloxone
D-Ticlopidine and Aspirin
A- Azithromycin
Which antibiotic causes QT prolongation? B- Doxycycline
C- Amoxicillin
A- Quercetin
A patient has experienced signs of toxicity including insufficient urination,
B- Furosemide
Which mediation is the cause?
C- Digoxin
A. Pregabalin
A 6-year-old boy is brought with complaint of uncontrollable spasm-like
B. Haloperidol
movements and excessive eye linking for four months. Chronic motor
C. Methadone
disorder diagnosed. Which of the following is the best management?
D. Chlorzoxazone
A. Ramelteon
Patient was prescribed hypnotic anxiolytic and is to be taken before
B. Zolpidem
bedtime, when she wakes up – she is still drowsy, what is the most likely
C. Diazepam
causative drug?
D. Midazolam
A. Give dose of 100 mcg and check levels
after 6-8 weeks B.Give dose of 150 mcg
and check levels after 6-8 weeks
A patient is taking 50 mcg levothyroxine and TSH levels are still high, what
C.Give dose of 25 mcg and check levels
adjustment should be made to treatment?
after 6-8 weeks
D. Give dose of 75 mcg and check levels
after 6-8 weeks
A. 5 mg Q4-6h
An 88-year-old patient takes 60 mg oral morphine for pain, and they want
B. 10 mg Q4-6h
to add short-acting morphine for breakthrough pain, what is the suitable
C. 20 mg Q4-6h
PRN dose?
D. 30 mg Q4-6h
A.stop antibiotic
A patient receiving levofloxacin bolus injection experienced sensitivity B.give one push
reaction, what is the best action? C.Given 45 mins
D.Give 90 mins
Patient came for dry cough medicine, headache, and other symptoms. She A. Refer to healthcare clinic.
hasn’t been to the clinic in months. She’s taking Captopril, metformin, etc. B. Tell patient it’s because of Captopril.
What’s the best intervention? C. Give the patient Guanfacine.
A. Salmeterol
Case about COPD and he asked about the drug that affect the prognosis of B. Salbutamol
the disease? C. Corticosteroid
D. Ipratropium
A. Leflunomide
Woman taking Methotrexate but radiology scan show progressive joint
B. Hydroxychloroquine
deformation which drug should we add?
C. Etanercept
Type 2 DM on insulin glargine and Metformin 750mg XR twice daily his
A. increase glargine dose
fasting glucose is normal but postprandial is elevated (281) what is the
B. Increase Metformin
best intervention?
C. Stop Metformin and switch to another
drug
D. Continue on the same regimen
A. Change the antibiotic
A patient taking Vancomycin 1gm Q12H, infusion rate 1gm/hour he B. Decrease the dose
developed skin rash and redness what is the best course of action? C. Increase the infusion rate
D. Decrease the infusion rate
A. Infection in which host cell dies due to
interference with normal cellular function.
B. Infections in which there is uncontrolled
growth or immortalization of the host cell.
C. Infection in which virus persists within
Which of the following viral infections best describes chronic infection?
the host cell but new virions are not
synthesized.
D. Infection in which virus persists within
the host cell and continues to replicate the
virions
A 16-month-old baby with bronchopulmonary dysplasia is admitted
A. Cefuroxime
Pediatric Intensive Care Unit with respiratory distress requiring in (see lab
B. Palivizumab
results) Heart rate 86/min, Respiratory rate 30/min, Temperature 38.6°C,
C.Corticosteroid
Test Result, Nasal swab Positive for respiratory syncytial virus, What is the
D. Intravenous fluids and supportive care
best management?
A. Amoxicillin: 500 mg PO q8h for 10 days
A 57-year-old man who presents to an urgent care clinic with complains of B. Azithromycin: 500 mg PO once daily for
2- day history of high fever, muscle/bone ache, headache and loss of 3 days
appetite. He has been in his usual state of health and reports close C. Amoxicillin-clavulanic acid: 1 gm PO
contacts of a sick friend with flu recently. Which of the following would beq12h for 1 week
the best management? D. Support treatment. (e.g., fluids, rest,
antihistamine, paracetamol)
A. Codeine 10 mg 4 times daily
A 10-year-old boy is brought with complaints of fever sore throat and non-
B. Guaifenesin 50 mg 4 times daily
productive cough for 1 day. Which of the following is best regimen for
C. Dextromethorphan 15 mg twice daily
non- productive cough?
D. Dextromethorphan 30 mg twice daily
A 76-year-old cardiac male patient was transferred to the hospital for
palpitation with a heart rate of 177 bpm. He is a heavy smoker, and his A. 1
past medical history includes hypertension, hyperlipidemia and coronary B. 3
artery disease. His medications include aspirin, clopidogrel, carvedilol, C. 4
valsartan and atorvastatin. He was diagnosed as having fibrillation. Which D. 6
of the following is the CHA2DS2-VASc score for this patient?
Patient is recently diagnosed with depression and taking citalopram and A. “It’s ok it’s depression”
told you that he doesn’t want to live, and the medication is doing nothing, B. “You’re not garbage”
how you will response? C. “You’re strong and young”
D. “How long have you had these
symptoms”
A. Tadalafil after 4 hours of taking
tamsulosin
Doctor wants to add a PDE-5 inhibitor, but patient is already taking B. Tadalafil and tamsulosin together.
tamsulosin. What instruction should you give to patient? C. Vardenafil after 4 hours of taking
tamsulosin.
D. Vardenafil and tamsulosin together.
A. Haloperidol
B. Risperidone
Patient case experiencing suicidal thoughts, withdrawal from society and C. Clozapine
angry? (The only FDA-approved medications for
social anxiety disorder are sertraline,
paroxetine, and venlafaxine.)
A. Warfarin 5 mg
A traveling a lot and had provoked DVT, his age is 45 and weight is 72kg his B. Apixaban 2.5 mg
creatinine level is 10 or 9 (given in mmol/L), which is the best option? C. Enoxaparin 40 OD
D. Enoxaparin 70 BID
A. LMWH 40 mg daily.
A patient has received PCI and wants prophylaxis for VTE. What should B. LMWH 1 mg/kg.
you give? C. Aspirin 81 mg per day.
D. Clopidogrel 75 mg per day.
A. Warfarin (Target INR 2.5 – 3.5)
A patient with mechanical heart valve requires prophylactic anti- B. Enoxaparin 40 mg
coagulation. What is the suitable treatment? C. LMWH
D. Apixaban 2.5 mg BID
A- take the dose 30 min to 60 min before
first meal for hypothyroidism
Simple instructions to give patient prescribed levothyroxine?
B- take the dose 30 min to 60 min before
first meal for underactive thyroid
A- Give PPI
B- Separate between doxycycline and the
Doxycycline with other topical agent for acne, he came to hospital
topical agent
complaint from heartburn and nausea what should you do?
C- Drink plenty of water with doxycycline
and take it before bedtime
A. Apply cream at night and take
doxycycline in the morning.
A patient prescribed doxycycline and acretin cream. The patient is B. Take doxycycline before bed with a full
experiencing esophageal irritation. Which advice should the patient have glass of water.
received to prevent this? C. Do not apply cream and take doxycycline
at the same time.
D. give patient omeprazole.
A- Increase the dose to 82 mg
B- Increase the dose to 107 mg
Pediatric patient was given vancomycin 75 mg and his trough level was 6, C- Increase the dose to 110 mg
what should you do to reach trough of 10? D- Increase the dose to 120 mg (Current
dose * (target level/measured level) = new
dose 75*(10/6) = 125 mg)
A. 6 Months
What is the optimal duration of aspirin to reduce the risk of colorectal B. 2 years
cancer? C. 10 years
D. >20 years
A. Increase dose of phenytoin
A patient with fatigue, anuria, hypotension, and low albumin is on B. Stop phenytoin because level is high
phenytoin and frusemide, what should you do? C. Stop phenytoin because albumin is low
D. Stop furosemide (B/C Anuria)
A. increase dose
Child with meningitis on vancomycin every 24 hrs with trough level 7 after B. increase frequency
two days. What is the best course of action? C. stop vancomycin
D. keep the same
A- 2 months
B- 3-4 months
What is the washout period of fluoxetine?
C- 5-6 months
(Fluoxetine washout period is 5 week)
A. Zolpidem
What treatment is recommended for patient with insomnia? B. Lorazepam
C. Diazepam
A. Ibuprofen and Pseudoephedrine
Patient with heart failure and runny nose, cough, sneezing, and pain. B. Paracetamol and Pseudoephedrine
Which treatment should be given? C. Ibuprofen and Xylometazoline
D. Paracetamol and Xylometazoline
A. Losartan
Hypertensive patient on ARB and CCB, and his BP not controlled; what
B. Spironolactone
medication you will add?
C. Indapamide
A. Donepezil
Patient with severe dementia and symptomatic relief was ineffective, what
B. Memantine
drug should be added?
C. Rivastigmine
A. Nitrofurantoin
B. Fosfomycin
Which of the following is the choice for a woman with severe UTI C. Trimethoprim/sulfamethoxazole
infection? D. Ciprofloxacin
((Severe or pyelonephritis use ceftriaxone
or cefazolin))
Pharmaceuticals
Mixing of substances to form a
What is the definition of miscibility?
homogenous solution without precipitates.
Prolongs half-life (acts as inactive estradiol
What is the benefit of conjugate estrogen compared to non-conjugate
reservoir that is slowly converted to active
estrogen?
estrogen).
Ointment (used for very dry,
What is the best base for dry skin?
chapped/cracked skin).
What is the function of the excipient Aspartame? Artificial Sweetener
Acacia, starch/cellulose paste, sucrose
Which of the following is a binder agent? syrup, compressible sugar, xanthan gum,
gelatin.
What is the advantage of transdermal delivery systems? Deliver drug at a controlled rate.
Which drug formulation should be stored in the refrigerator? Suppositories
Which device should be dispensed with an oral liquid medication to child? Oral syringe or spoon
What is the function of PEGylation of medications? To increase half-life
Aspirin is an acid with pKa of 3.5, which is the % ionization in the
99.9%
intestines (pH= 7.4)?
What is the best pH for absorption of acidic drug with 3.2 Pka ? pH 1 (predominantly unionized)
pH 1 (acidic drugs are predominantly
An acidic drug with pKa of 3.5, what is the best pH for it to be absorbed?
unionized at pH below their pKa)
What type of agents are suspending agents? Hydrophilic colloids
What is the role of bentonite? Suspending agent
A medicine contains water and olive oil, what type of formulation? Emulsion
Which USP chapter can be used for information of chemotherapy
USP 800
preparation?
What is USP chapter 797 used for? Sterile Production
What is the risk level of contamination when transfer a sterile medication
Low-risk level
by a sterile needle to a sterile package?
Patient with TPN and he takes tablet drug, what is the appropriate sustained release tablets if crushed will
answer? cause harm to patient
To increase penetration of drug but can be
Why are hypertonic solution used in intramuscular injections?
painful.
What type of water is used in TPN/IV admixture? Water for injection
A drug has a half-life of 5 hours, how much of the drug will be cleared in
97%
24 hrs?
Which of the following is a non-aqueous formulation? Elixir
A. Diluent
Why is sodium alginate added to formulation? B. Disintegrant
C. Binder
A. Low dose, high bulk density
Which drug properties is suitable for hard capsules?
B. High dose, low bulk density
A. Reduce Flowability
B. Reduce Stability
What is the purpose of granulation of drug particles? C. Reduce Surface Area
D. Reduce dust from compounding toxic
chemicals.
A. Formulate as solution ready to use
A drug has pKa of 6-7, what would be the best formulation of this drug to
B. Reconstitute with WFI before use
make it have longest shelf-life?
C. Reconstitute with solution with pH 6
A. Faster onset of action
Transdermal Patch advantage for lipid soluble drugs? B. Increased bioavailability
C. Near constant rate as IV infusion
A. A solution
Medication order is given to the pharmacist (see table) (Olive oil 60 ml,
B. A ointment
Vitamin A 60.000 U, Water 120 ml, Sig 15ml t.i.d) What is likely to be the
C. An emulsion (contain oil and water)
final form of preparation?
D. A suspension
A. IV drugs transmits the highest risk of
infection
B. IV drugs are associated with lowest risk
Which of the following statements best represents the intravenous (IV) of undesirable side effects C. IV solutions
medications in the hospital facilities? must be hypotonic to ensure compatibility
with blood osmolality
D. IV solutions must be hypertonic to
ensure compatibility with blood osmolality
A. Sterile water
Which of the following type of water should be used for reconstitution of B. Distilled water
parenteral preparations? C. Sterile water for injection
D. Bacteriostatic water for injection
A. Hydrophilic, to inhibit agglomeration
B. Hydrophobic, to inhibit agglomeration
A suspended agent is important for preparing an oral suspension. Which C. Hydrophilic, to prolong gastric emptying
of the following is the characteristic of this suspending agent? time
D. Hydrophobic, to prolong gastric
emptying time
121 C for 30 mins
210 C for 1hr
121 C for 15 mins
Pyrogen present in containers can be destroyed by heating the container 180 C for 4 hrs
at? (Depyrogenation (FDA):
- 650 C for 1 minute
- 250 C for 45 mins
- 180 C for 4 hrs)
In a quality check, a tablet was thrown from a height using a rotating A. Dissolution test
drum. What is the name of this test? B. Hardness test
C. Disintegration test
D. Friability test
A. Antioxidant
B. Suspending Agent
Why is boric acid added to the formulation?
C. Buffer
D. Solvent
Calculations
A pharmacist was asked to produce 10 x 1 gram suppositories (each
containing 200 mg of drug X). Given that the displacement value Is 4.6, 9.57g of base
how much base is required?
A pharmacist needs to make a solution with concentration 0.5% w/v. The
available stock solutions are 120 ml of 20 mg/5mL solution and 240 ml of
24 mL of the 50 mg/5mL solution
50 mg/5ml solution. How much of the second solution should be added to
the first solution to provide the desired concentration?
Hepatic elimination ratio*hepatic blood
flow = hepatic clearance OR rate of hepatic
How is hepatic clearance calculated?
elimination*drug concentration = hepatic
clearance
Number of death cases per year/total
How is mortality calculated?
population * 100
Number of people with disease/total
How is prevalence calculated?
population*100
peak-trough levels within one dosing
What is the definition of fluctuation (pharmacokinetic)?
interval at steady state (expressed as %).
What is the equation for drug elimination constant in first order kinetics?
How is clearance measured in an adult patient? Cockcroft & Gault equation
1 gallon =3800 mL thus 10 mEq/L = 38
Calculation switching between mEq per liter to mEq per gallon?
mEq/gallon
The doctor prescribed 25 mg/m2 of Drug for a child weighing 18 kg and
16 mg
his height is 82. Calculate the dose?
What is the AUC of IV Ceftriaxone considering half-life is 6 hours and initial
259.7 mg.h/L
concentration is 30 mg?
Sun protection factor (SPF) = Time-to-burn
Calculation of SPF? (with sunscreen) / Time-to-burn (without
sunscreen)
The strength of attraction between drug
What is the definition of affinity?
and receptor.
The patient took 100 mg of drug with half-life of 5 hours. How much of
97%
the drug will be eliminated in 24 hours?
A patient is prescribed ‘Phenytoin 100 mg dTd 50, 1 p.o.q.d’. The only 50 tablets * 100 mg * 1 p= 5,000 mg / 600
available strength is 600 mg. How many tablets is needed? mg = 9 tablets (8.33)
Drug A and Drug B both cost 60 thousand dollars to manufacture, 10,0000
of each drug were produced. Drug A was sold for 40 dollars and Drug B (40,000 + 80,0000) – 60,000 = 60,000 SAR
was sold for 80 dollars. How much profit was made?
0.87 or 87%
Hepatic Clearance = Hepatic perfusion *
Calculate bioavailability if hepatic blood flow is 1500 ml and hepatic Extraction fraction Bioavailability = 1 –
clearance is 200 ml/min? Extraction fraction
200 ml/min ÷ 1500 ml = 0.13
1 – 0.13 = 0.87
Pharmacist received prescription of hydrocortisone 2% 60g cream. The
available strength is 1% and 2.5% . what is the amount of both 40g of 2.5% and 20g of 1%
concentration you need to prepare 2%?
0.46 g of NaCl
A patient is prescribed atropine 1% (E=0.13) in purified water and normal 0.9% NaCl*60mL = 0.54 g of NaCl
saline up to 60mL. What is the amount of NaCl required for it to become Atropine 1% in 60 ml = 0.6 g * 0.13 = 0.078
an isotonic solution? g equivalent of NaCl 0.54 g – 0.078 g=
0.462 g of NaCl
A hospitalized patient with hypertension is prescribed nicardipine HCl IV
infusion at a dose of 5 mg per hour for 4 hours. Nicardipine HCl is
8 mL
available as 10 ml vial containing 25 mg of nicardipine HCl. How many ml
should be used?
Child weighing 25 kg is prescribed acetazolamide 2.5 mg/kg every 12
hours. Acetazolamide is available in a liquid dosage form with strength of 2.5 mL
25 mg/ml. How many ml would patient be administered per dose?
A 20 ml vial of penicillin-G potassium containing 2,000,000 units is
available. Each mg of the drug is equivalent to 1600 units. How many 62.5 mg/mL
milligrams of penicillin-G potassium are there in one ml of the solution?
Convert patient weight from pound to kg. weight in pounds/2.2 = weight in kg
A 500 mg dose of aspirin was given intravenously to a patient produced an
initial concentration of 100 mg/L. if the half-life of the drug is 6.93 hours, 0.5 liters/hr
what would be the total body clearance?
Convert SCr in micromoles/L to mg/dL by
Calculating creatinine clearance and serum creatinine (SCr) is given in dividing it by 88. Then use the Cockcroft-
micromoles/L. gault equation to calculate creatinine
clearance.
A-Take 1 tablet once daily (21 tablets)
The following prescription is received in a hospital pharmacy: Losartan 25
B-Take 2 tablets once daily (42 tablets)
mg, Sig: Take 1 tab BID, Duration: 3 weeks, Which of the following
C-Take 1 tablet twice daily (42 tablets)
instructions will be in prescription label?
D-Take 2 tablets twice daily (48 tablets)
A 35-year-old individual is receiving 20 mg of propranolol hydro 4 times A- 2.5 mg
daily as a prophylaxis to migraine attack. The drug bioavailability is 25% B- 5mg
due to extensive first pass effect. What amount of the drug reaches the C- 7.5 mg
systemic circulation after dosing? D- 10mg
A-Molecular weight > 500 Da
B-Log P <5
Which of the following is related to Lipinski’s rule of 5?
C-H-Bond Donors <10
D-H-Bond Acceptors <5
Medicinal
How is amphotericin B metabolized? Hydrolysis
What type of breakdown occurs due to presence of water? Hydrolysis
What is the reaction that converts salicylic alcohol to salicylic acid? Oxidation
What reaction is involved in metabolism of prednisone to active form
Oxidation (via : dehydrogenase)
prednisolone?
Which is the most common cause of drug instability? Oxidation
Metabolism of Catecholamine? Oxidative deamination by MAO
What type of reaction causes the conversion of amine (NH3) to a carbonyl Oxidative deamination (catalyzed by MAO
(C=O) in a catecholamine molecule? enzyme)
What type of reactions is the conversion of NAD to NADH? Redox Reduction reaction
Degradation of aspirin via hydrolysis
A patient left her aspirin pills in the bathroom, what will likely happen to
(bathroom is a warm and humid
aspirin?
environment)
Which of these ACE inhibitors structures are active? Captopril and Lisinopril
What is the least dominant form of histamine? When both amine groups are protonated
What is the property of Carboxylic group (COOH)? Acidic
What is the property of Amine group (NH3)? Basic
What is the property of Alkyl group (CH3)? Neutral
Demethylation of the side-chain tertiary
What is the structural change in SNRI that forms NRI?
amine to secondary amine
which part of the structure of ranitidine is responsible for its H2-receptor
Furan ring
inhibitory activity?
What type of bond does paracetamol make? Hydrogen bond
What is a chemical compound during drug discovery that has
pharmacological or biological activity against target and thought to be A lead compound
therapeutically useful?
At which pH will aspirin be most stable? pH <2
A female patient has ulcerative colitis and dysmenorrhea, based on the
Mefenamic Acid
given structures, which treatment should be given?
Alcohol group: H-bond, ester: hydrolysis,
structure of a drug’s pharmacophore and asks about the interactions of
Alkene: hydrophobic interaction,
different functional groups?
Carboxylate: ionic interaction
Tyrosine, L-DOPA, Dopamine,
What is the intermediate of epinephrine? Norepinephrine are all considered
intermediates of epinephrine
(Oseltamivir carboxylate) Ester hydrolysis
What is the active form of Oseltamivir?
to Carboxylate
What is the reason for the substitution of the ethylmethyl sidechains of an Improve binding to hydrophobic binding
ACEIs to a benzyl group? bucket.
Why are 2 atom linkers added to ramipril? To improve binding to hydrophobic pocket
What is the benefit of changing carboxylic acid to tetrazole in Losartan? Improved oral absorption
Which one between Daunorubicin (right) and mitoxantrone (left) has Mitoxantrone is less cardiotoxic because it
higher cardiotoxicity and why? lacks the sugar moiety.
The pyridine nitrogen, which makes it more
What is the function of substituents on the pyridine ring of proton-pump
reactive with the cysteine residue of the
inhibitors?
proton pump.
Antihistamines increase pH of stomach and
What is the interaction between antihistamines and proton-pump
low pH (acidic) is required for activation of
inhibitors?
PPIs.
Which of the following structures is aspirin? ____
Which one is a DHP CCB from the following chemical structure? ____
Which amino acid contains an imidazole ring? Histidine
What is the impact of modifying the isopropyl group of isoprenaline with
Improve binding to target
benzene?
Ketal formation between hydroxyl group (6
What is the reason for erythromycin instability in acidic conditions?
and 12) and ketone at position 9.
What is the pharmacophore of glimepiride? Sulfonamides
Which one is a 5 generation cephalosporin beta-lactam? Ceftaroline
Mirror images of each other but they
How can you distinguish enantiomers? cannot be superimposed upon one
another.
What are two molecules have the same molecular formular and sequence
of bonded elements, but they are non-mirror images and are non- Diastereomers
superimposable?
Active form of diclofenac? 4-hydroxydiclofenac generated by CYP2C9
What time of reaction convers Prontosil to Sulphonamide? Metabolic reduction
Metoprolol undergoes Phase II conjugation metabolism with glucuronide.
Hydroxyl group
Which group undergoes glucuronidation?
Which chemical reaction is involved in conversion of morphine to heroin? Reductive Acetylation
Which chemical reaction is involved in conversion of morphine to
Reductive Methylation
codeine?
Which chemical reaction is involved in conversion of codeine to
Oxidative Demethylation
morphine?
A- Glycolysis
What type of reaction converts alanine to glucose? B- Alaninolysis
C- Oxidative deamination
A.Endothermic
What type of reaction is the conversion of malate to oxaloacetate (Gibbs B.Exothermic
free energy +29.7 kJ/mole)? C.Endogenic
D.Exogenic
A. Etidocaine
Ropivancaine is an important local anesthetic. The figure shows the
B. Bupivacaine
chemical structures of ropivacaine and 4 other local anesthetics (see
C. Mepivacaine
image). Which of the following is the bioisoster for ropivacaine?
D. Levobupivacaine
A. Isomers
Which of the following is the most appropriate term used to call the two B. Epimers
conformations of the thalidomide? C. Enantiomers
D. Diastereomers
Galactose and D-Mannose are monosaccharide sugar. They have the same A. isomers
number of carbons and the same functional group, but they differ in their B. epimers
configurations. This reflects their pharmaceutical uses (see image). Which C. anomers
of the following types of stereo-isomerization do these sugars represent? D. Diastereomers
Glucose and D-Mannose are monosaccharide sugar. They have the same A. isomers
number of carbons and the same functional group, but they differ in their B. epimers
configurations. This reflects their pharmaceutical uses (see image). Which C. anomers
of the following types of stereo-isomerization do these sugars represent? D. diastereomers
A. Benzene sulfate urea
A structure of the glibencalmide (glyburide). Which of the following B. Benzene sulfinyl urea
pharmacophore does Glibencalmide (glyburide) contain? C. Benzene sulfhydryl urea
D. Benzene sulphonyl urea
A. Activity has increased due to increase
molecular weight.
B. The analogue has an altered pKa.
C. The extra phenol ring in the analogue is
Isoprenaline is an adrenergic agonist. The isoprenaline analogue shown
interacting with an addition binding region
below demonstrated that can be drawn?
in the binding site.
D. The extra substituent is protecting the
terminal methyl group of isoprenaline from
metabolic reactions.
A. Ethosuximide
Question describes symptoms of a tonic-clonic seizure (does not say the
B. Diazepam
diagnosis), what is the suitable treatment (gives you the structures and
C. Fosphenytoin
names)?
D. Gabapentin
A drug structure with the following labelled groups: amine group (A), 1. A is basic, B is basic, C is acidic.
pyridine (B), and carboxylic acid (C) and asks about their properties? 2. A is basic, B is neutral, C is weakly acidic.
Photos
Identify the pharmacophore of
fluoroquinolone

Pharmacophore of Proton Pump Inhibitors

What is the function of substituents on the


pyridine ring of proton-pump inhibitors?
Electron donating substituents (R1 = OR,
R2 and/or R3= CH3) on the pyridine ring
increases the % ionization of PPI and
increases nucleophilic character of the
pyridine nitrogen, which makes it more
reactive with the cysteine residue of the
proton pump.

Based on the chemical structures, which


steroid can be taken via inhalation?
Fluticasone

A female patient has ulcerative colitis and


dysmenorrhea, based on the given
structures, which treatment should be
given?
Mefenamic Acid

What reaction is involved in metabolism of


prednisone to active form prednisolone?
Which of the following structures is
Ethosuximide?

Which of the following structures is


aspirin?

Which one between Daunorubicin (right)


and mitoxantrone (left) has higher
cardiotoxicity and why?
Mitoxantrone is less cardiotoxic because it
lacks the sugar moiety, which makes
daunorubicin more prone to mitochondrial
redox cycling in the cardiomyocytes. This
results in generation of reactive-oxygen
species (ROS).

What is the benefit of changing carboxylic


acid to tetrazole in Losartan?
Improved oral absorption

What is the impact of modifying the


isopropyl group of isoprenaline with
benzene?
Improve binding to target

Active form of diclofenac?


4-hydroxydiclofenac generated by CYP2C9
PK equations

From the table above, what is Drug A?


Direct Renin Inhibitor (e.g., Aliskiren)

From the table above, what is Drug C?


Angiotensin receptor blocker (e.g.,
Valsartan)

Calculating dosing interval of Gentamicin


using a Hartford Nomogram

Methicillin sensitive strains


Strategies for Preventing Medication Errors
and Quality Improvement

Medication Errors categories

Lipinski’s rule of 5 for is to determine if a


chemical compound with a certain
pharmacological or biological activity has
chemical properties and physical
properties that would likely make it an
orally active.

Efficacy and potency of a drug


Which drug has shorter half-lif on
concentration-time graph?

Which drug has higher bioavailability on


concentration-time graph?

Type of examination?
kernig sign

Single nucleotide polymorphisms (SNPs)


are the major form of genetic variations
Treatment of hyperkalemia

COPD classification based on FEV1% gold


standard.

At which pH is the drug most stable (see


Stability-pH plot below) ?
Which amino acid is acidic?

type of monoclonal antibody?

cost category?

vitamin d pathway

‫ أسال هللا لكم التوفيق والتيسير والنجاح‬،‫تم بحمد هلل‬

You might also like